Cleary Exam 2 :(

Pataasin ang iyong marka sa homework at exams ngayon gamit ang Quizwiz!

A nurse caring for a caseload of patients is dlegating care to the nursing assistive personnel (NAP). After providing instuctions, what care should the nurse delegate to the NAP ? SATA A. administering a suppository to a patient who has constipation B. Monitoring the intravenous fluid rate of a patient who has diarrhea C. Documentation of urine output from a patient who is NPO waiting for surgery D. Documentation of fluid intake for a patient who is prescribed a clear liquid diet

B,C,D

A nurse reviews laborator data of a patient and notes the hemoglobin has been decreasing for several months. What diagnostic test should the nurse anticipate the provider will order? A. Swallow study B. Stool for guiac C. Ultrasound of the abdomen D. Abdominal x- ray

B-- Guiac means blood in stool, occult blood has hemoglobin, therefore if the PT is pooping occult blood for several months they may have low hemoglobin

A nurse is teaching a patient with a new ileostomy about incorporating preventive strategies at home. To prevent excoriation and breakdown of the peristomal skin, the nurse should instruct the patient to A. Apply hydrocortisone cream to the skin when changing the appliance. B. Empty the pouch when it is no more than half full. C. Wash the peristomal skin frequently with deodorizing soap and water. D. Choose a time shortly after a meal for replacing the pouch.

B. Empty the pouch when it is no more than half full. Waiting until the pouch is more than half full increases the risk of leakage. Ileostomy effluent is irritating to peristomal skin, so patients should replace the pouch when it is one-third to one-half full.

A nurse is providing preoperative teaching for an older patient who has diverticulitis and is scheduled for a creating for a double barrel colostomy in the sigmoid colon. Which of the following instructions should the nurse include in the teaching? A. Irrigate both stomas periodically to promote drainage. B. Tape a dry gauze pad over the distal stoma to collect drainage. C. Change the proximal stoma's appliance every other day. D. Expect liquid stool to drain from both stomas.

B. Tape a dry gauze pad over the distal stoma to collect drainage. The distal stoma (also called a mucous fistula) secretes mucus; it does not drain feces. A dry gauze dressing is usually sufficient.

A nurse admits a patient who reports a 15 pounds weight loss over the last two months. The nurse calculates BMI of 16.5 kg/m2. What laboratory values should the nurse review to correlate with this data ? A. Serum sodium B. Serum albumin C. Blood urea nitrogen D. Blood glucose

B. albumin has a long half life is 18 days. She has lost 15 pounds in 2 months ! To check the nutrition

A client has impaired swallowing. Which of the following interventions is most appropriate for the RN to use? A. Provide straws for use B. Encourage the client to tuck their chin with swallowing C. Feed client while in supine position D. Encourage chewing on the affected side after stroke

Boy

A female client who has had three vaginal deliveries in 5 years and leaks urine when she laughs, will like be diagnosed with what type of urinary elimination problem? A. Urinary urge incontinence B. Urinary stress incontinence C. Insufficient ADH secretion D. Urinary retention

Boy

The RN is assigned to a female client with the nursing diagnosis of imbalanced nutrition less than body requirements. Which of the following could they expect to find on assessment? A. A waist to hip ratio of 0.8 B. A BMI of 16.5 C. A BMI of 19 D. An increased skin fold measurement

Boy

A nurse admits a frail older pt who has fecal impaction from a long term care facility. What diagnostic test should the nurse anticipate? A. Sigmoidoscopy B. Fecal occult blood test C. Abdominal flat plate X-ray D. Ultrasound

C

A nurse admits a pt who has an ILEOstomy. When assessing the stoma and output, what finding should the nurse expect? A. Hard dry stool in the bag B. 2 separate stomas in the upper quadrants of the stomach C. Continuous liquid stool in the bag D. Soft formed stool in the bag

C

A nurse admits a pt who reports unintentional 15 lbs weight loss over the last 2 months. The nurse calculates a BMI of 16.5. What lab values should the nurse review to correlate with this data? A. BUN B.Serum sodium C. Serum albumin D. Blood glucose

C

A nurse assesses and older pt who reports fewer bowel movements as they aged compared to when they were younger. What should the nurse suggest? A. Twice weekly serving of yogurt B. A glass of orange juice at lunchtime C. Adding a few prunes to their cereal at breakfast D. An afternoon snack

C

A nurse completes the Braden scale on a patient who scores 10 with deficit areas of mobility and activity. When planning care, what intervention should the nurse include in the plan? A. Provide a bedside commode B. Reposition the patient every four hours C. Ambulate the patient a short distance every two hours D. Maintain the head of the bed elevated to 60 degrees during the day

C

A nurse examines a stage III pressure injury on the sacrum of a 45 year-old woman and notes a large amount of exudate soaking the periwound. What factor should the nurse recognize will negatively affect skin integrity and healing? A. Location of the pressure injury B. Age C. Amount of exudate D. Gender

C

A nurse is instructing a nursing student how to set up a sterile field. What action if done by the student should the nurse identify will contaminate the field? A. The student opens the first flap away from self B. The final flap of the dressing pack is opened towards the student C. The student places the sterile package on a wet surface D. The student closes the door to the area where the dressing will be done

C

A nurse is planning care for a patient who has nausea and loss of appetite after recieving chemotherapy. What intervention should the nurse include in the plan ? A. Keep the patient NPO B. Ensure suction is set up C. Provide bland meals and snacks D. Maintain the head of the bed elevated for one hour after each meal

C

A nurse is planning care for a pt who reports involuntary loss of urine when coughing or laughing. What intervention should the nurse include in the plan? A. Provide a bedside commode B. Apply heat to the lower abdomen to relax muscles C. Teach pelvic floor strengthening exercises D. Initiate scheduled toiling program

C

A nurse working in a clinic takes a health hx from a young woman who reports frequent and burning urination for the last 2 days. What test should the nurse anticipate the provider will order? A. Serum creatinine B. BUN C. Urinalysis D. Specific gravity

C

A nurse is applying a condom catheter for an older adult pt who is uncircumcised. Which of the following is an appropriate step in the procedure? A) Stretching the catheter along the length of the penis B) Securing the catheter with adhesive tape C) Leaving a space between the penis and catheter's tip D) Repositioning the foreskin after application

C A - A condom catheter is rolled smoothly over the length of the penis, not stretched along its length. B - Adhesive tape does not expand with changes in the size of the penis (possibly impairing blood flow) and is painful to remove. Therefore, it should not be used to keep a condom catheter in place. C - A space of 2.5 to 5 cm (1 to 2 in) should be left between the tip of the penis and the end of the catheter. This space helps prevent irritation of the tip of the penis and allows full drainage of urine. D- No manipulation of the foreskin is required during the application of a condom catheter.

A nurse is caring for a pt who is 8 hours post op abdominal surgery and their diet order has been changed from NPO to clear liquid. What care can the nurse delegate to the NAP? SATA A. Monitor IV fluid rate and check infusion pump B. Teach the pt about appropriate fluid choices C. Assist the pt into a Semi-Fowlers position to drink apple juice D. Provide oral care after the pt has consumed jello E. Examine the amount and type wound drainage

C D

A nurse reviews a patient's chart and finds documentation of deep tissue injury on the patient's sacrum. When examining the patient what findings should the nurse expect? Select all that apply: A. Exposed bone B. Stringy yellow tissue C. A discolored blister D. Intact skin E.Black leathery tissue

C D(eez nuts)

While a nurse is administering a cleansing enema, the patient reports abdominal cramping. Which of the following is the appropriate intervention? A) Measure the patients vital signs B) Notify the primary care provider C) Lower the enema fluid container D) Stop the enema instillation

C) Lower the enema fluid container

A nurse assesses an adult patient whose BMI is 17 and reports recent weight loss over the last couple of months. What action should the nurse take ? A. Assess the patients gag reflex B. Ask the provider to order a complete blood count C. Instruct the patient to complete a 3 day food diary D. Review the patients current medication list

C. A three-day food diary will give the provider more information as to what the PT is eating

A client is preparing for a diagnostic test for bowel elimination issues. Which of the following tests will the RN instruction the patient that they will be sedated for? A. Barium Enema B. Abdominal Ultrasound C. Colonoscopy D. Abdominal CT Scan

C. Colonoscopy

A nurse is caring for a patient who has an ileostomy and is eating a regular diet. When assessing the patient, what is an expected finding ? A.Formed stool in the bag B. Hard clay colored stool in the bag C. Liquid stool in the bag D. Semi-formed stool in the bag

C. Ileostomy is found in the small intestine which does not absorb water SO the stool will be watery.

The RN is developing a plan of care for a client with altered bowel elimination: constipation who is on bed rest. Which of the following is an appropriate nursing intervention for this client? A. Encourage water intake of 1,000 mL daily B. Encourage fiber intake of 10-20 grams daily C. Provide range of motion by bending clients knees as if riding a bicycle D. Encourage beans and broccoli as increased gas will help push out hard stools

C. Provide range of motion by bending clients knees as if riding a bicycle

A nurse assesses an older patient who reports fewer bowel movements as they aged compared to when they were younger. What should the nurse suggest ? A. twice weekly servings of eggs B. An afternoon snack of crackers and cheese C. Adding raisins to their oatmeal at breakfast

C. Raisins make you poop

A nurse is instructing a patient how to collect stool for occult blood. What instructions should the nurse provide ? A. Wash your rectal area with soap and water before collecting the specimen B. Wear sterile gloves when collecting the specimen C. Eliminate red meat, fish, chicken from your diet for 2 days before collecting the specimen D. Urinate before and after collecting the specimen

C. Red meat can give false positive

A nurse is providing preoperative teaching for a patient who has colon cancer. The surgeon informed the patient his entire large intestine and rectum will be removed. The nurse should explain the type of ostomy he will have is A. A cecostomy. B. A loop colostomy. C. An ileostomy. D. A descending colostomy.

C. a ileostomy After removing the entire large intestine and the rectum, the surgeon will create a cecostomy to divert feces from the small intestine to the abdominal surface and into an ostomy pouch.

The RN is assigned to a client with a bowel diversion. Which of the following would the RN anticipate having the least formed stool? A. Sigmoid colostomy B. Descending colostomy C. Ileostomy D. Ascending colostomy

Cat

The RN is preparing a client for an enema. Which of the following is the appropriate position to have the client in? A. Prone B. Supine C. Left side with right knee bent D. Standing and bending over bed

Cat.

A nurse assesses a pt who appears anxious (same tbh). RR is 32bpm and the pt reports their fingers and hands are numb and they feel lightheaded. What data would correlate with this? A. Normal CO2 levels B. Normal oxygen levels C. Decreased heart rate D. Increased heart rate

Cleary said B in her lecture NP i know said it has to be D because tachycardia is the main symptom of anxiety. They would not have normal oxygen levels due to such a high respiratory rate. They would have low CO2 in the blood due to the excess breathing. And decreased heart rate is the filler answer and does not correlate with anything in the question.

A nurse assesses a patient and notes a deep ulcer with necrosis of subcutaneous tissue on the patient's heel. How should the nurse document this finding? A. Deep tissue injury B. Unstageable pressure injury C. Stage II pressure injury D. Stage III pressure injury

D

A nurse is instructing a pt how to collect stool for occult blood. What instructions should the nurse provide? A. Wear sterile gloves when collecting the specimen B. Urinate before and after collecting the specimen C. Wash your rectal area before collecting the specimen D. Eliminate red meat, fish, and chicken from diet 2 days before collecting specimen

D

A nurse is preparing to administer a prepackaged enema to a pt. How should the nurse position the pt? A. Prone B. Semi-Fowler C. Supine D. Sims

D

A nurse is providing wound care and notes the wound drainage is thick and malodorous. How should the nurse document this type of exudate? A. Serosanguineous B. Purosanguineous C. Serous D. Purulent

D

A nurse is assessing a patient's indwelling urinary catheter drainage at the end of the shift and notes the output is considerably less than the fluid intake. Which of the following actions should the nurse take first? A) Irrigate the catheter. B) Assess for peripheral edema. C) Palpate for bladder distention. D) Milk the catheter.

D A - If the catheter is blocked, the provider might have to prescribe catheter irrigation; however, this is not the first action the nurse should take. B - Assessing the extremities for peripheral edema is an appropriate action, but it is not the first action the nurse should take. Palpate for bladder distention. C - Palpating the bladder for distention is an appropriate action, but it is not the first action the nurse should take. D - Output that is considerably less than intake is a sign that the catheter is blocked. The first action the nurse should take is to milk the tubing by squeezing then releasing the drainage tube, starting from near the patient and moving toward the drainage bag. This should dislodge any buildup of blood, pus, or sediment.

When providing perineal care for a female patient who has an indwelling urinary catheter, which of the following areas should the nurse cleanse last? A) The urethral meatus B) The labia minora C)The perineum D) The anus

D ----CLEAN TO DIRTY A - The urethral meatus is the first area the nurse should cleanse when giving perineal care. B - Cleansing the labia minora last would violate a basic principle of asepsis. C - Cleansing the perineum last would violate a basic principle of asepsis. D- The basic aseptic principle applicable to perineal care is to cleanse from the area of least contamination to the area that is the most contaminated. The anal area is typically contaminated with coliform bacteria and should be cleansed last.

A nurse is preparing an older adult patient for an enema. The nurseshould assist the patient to which of the following positions? A) Prone B) Dorsal recumbent C) Right lateral with both knees at chest D) Left lateral with the right leg flexed

D) Left lateral with the right leg flexed

A nurse is planning care for a patient who has difficulty swallowing after a stroke. What intervention should the nurse include in the plan ? A. Provide frequent oral hygiene B. Provide finger foods C. Encourage sips of fluid between meals D. Feed in small amounts

D.

A nurse admits a frail older adult from a long term care facility. When reviewing laboratory data, what test result would indicate a need for referral to the nutritionist ? A. Serum glucose 90 mg/ml B. Creatine 0.7 mg/dL C. Blood urea nitrogen 15 mg/dL D. Serum albumin 3.0 g/dL

D. Albumin levels should be between 3.5 - 5 g/dL

A nurse is replacing the ostomy appliance for a patient whose newly created colostomy is functioning. After removing the pouch, which of the following should the nurse do first? A. Measure the stoma. B. Cover the stoma with gauze. C. Remove the backing on the skin barrier. D. Cleanse the stoma and the peristomal skin.

D. Cleanse the stoma and peristomal skin. To facilitate the nurse's assessment of the stoma and peristomal skin, the Nurse must remove any effluent adhering to the area.

A nurse is teaching a patient how to apply an extended-wear skin barrier. Which of the following strategies should the nurse instruct the patient to use for maximal adherence? A. Use an oil-based lotion on the peristomal area. B. Apply the skin barrier while the skin is slightly moist. C. Leave the residue from the previous appliance on the skin. D. Press gently around the barrier for 1 to 2 minutes.

D. Press gently around the barrier for 1 to 2 minute. The pressure-sensitive tackifiers and heat-sensitive polymers of the skin barrier require adequate pressure (and warmth from the fingers) to ensure adherence.

A nurse is preparing to administer a prepackaged enema to a patient. How should the nurse position the patient ? A. Prone B. Semi-fowler C. Suspine D. Sims

D. Prone is on the stomach , semi fowler half up, supine is on back. Sims is right knee up and left leg don used for any rectal exams.

When teaching the parents of a toddler about feeding and eating the nurse should include what safety measure?

DO NOT offer the child raw veggies(raw vegetables, as well as hot dog, grapes, nuts, popcorn, and candy, have been implicated in choking deaths and should be avoided at least until the child is 3 years old

A client is complaining of acute constipation. Which of the following is the most appropriate question for the nurse to ask to aid in diagnosis? A. What color are your stools? B. What time of day do you typically have a bowel movement? C. How much fiber do you eat daily? D. How often do you typically have a bowel movement?

Dog

The RN is reviewing lab results for a 25 year old client The clients BUN is 25 mg/dL (elevated). The RN knows this may indicate all of the following except: A. Dehydration B. Impaired kidney function C. Increased protein intake D. Impaired liver function

Dog

deez nuts

HA GOT EEM

Which of the following strategies for enhancing the intake of healthful foods is appropriate for an adolescent?

Making the healthful food choices more convenient and available for the adolescent

A patient who is postoperative is experiencing abdominal distention and is having difficulty expelling flatus. The nurse should anticipate receiving an order from the provider for which of the following types of enemas? A) Cleansing B) Oil retention C) Medicated D) Return flow

Return Flow

A nurse caring for a patient who has a head injury and level of consciousness fluctuates. The provider orders a full liquid diet progressing to a pureed diet as tolerated. Before feeding what is essential that the patient undergo?

Swallowing examination (patients at risk for aspiration include those with a decreased level of consciousness. This patient has some periods of decreased alertness, thus a swallowing examination is essential to determine his ability to ingest food safely by mouth.

A nurse assesses a 45-year-old man who lives with wife. The patient is incontinent of urine and walks with a cane. What factor should the nurse identify will increase risk for developing a pressure ulcer? A. The use of a cane B. The urinary incontinence C. The patient's age D. The living situation

b

A nurse admits a patient who is lactose intolerant and reports watery stool 6-7 times over the last 24 hours. When developing the plan of care, what outcome should the nurse include ? A. Manage self-care within 24 hours B. Identify areas of weakness and assistance needed within 2 days C. Verbalize sense of control in current situations D. Reestablish and maintain normal pattern of bowel functioning

b? because it is timely ? OR D??

A nurse is likely to receive an order for urinary catheterization of a newly admitted patient who A) has a persistent urinary tract infection. B) has urge incontinence. C) is in the ICU for a gastrointestinal bleed. D) is incontinent due to cognitive decline.

c A - Urinary tract infections are treated with antimicrobial agents, increased fluid intake, and pain management, not urinary catheterization. B -Treatment options for urge incontinence typically include pelvic floor exercises, medications, and bladder retraining, not urinary catheterization. C - Precise measurement of urinary output is crucial for managing fluid balance in patients who are critically ill. D - This is a type of functional incontinence. Typical interventions include scheduled toileting and absorbent products. Catheterization would be a last resort for this patient.

While a nurse is teaching a patient how to replace her ostomy pouching system, the patient reports that removing the skin barrier is sometimes painful. Which of the following should the nurse suggest? A. Lift up on both sides of the skin barrier simultaneously. B. Release one corner of the barrier and pull it quickly over the stoma. C. Push the skin away from the barrier while removing it. D. Gently roll the barrier end-over-end across the stoma.

c. Push the skin away from the barrier while removing it. Pushing the skin away from the barrier helps prevent skin stripping, which can be painful and make the skin sensitive to the adhesive. If the patient is having difficulty with the initial release of the barrier, it may help if she starts in one corner and gently pulls it over the stoma while pushing the skin away from the barrier.

Which of the following tasks must be performed by the RN and cannot be delegated? A. Recording output from a urinary catheter in the client chart B. Inserting a foley catheter in an otherwise stable client C. Performing an abdominal assessment to determine the location of pain D. Toileting the client after every meal

cat

Which of the following dietary modifications should an adolescent engaging in sports implement

drink water before and after sports activities

A nurse is caring for a patient who has impaired swallowing due to a cerebrovascular accident. Which of the following interventions should the nurse use to assist the patient with feeding? THINK ABOUT POSITIONING

elevate the head of the bed to 45-90 degrees

How should a nurse describe mealtime for a pt who has a visual deficit?

identify the food location as though the plate were a clock (facilitates independence of eating).

To assess a patient for adequate swallowing the nurse should do which of the following?

place fingers on the patients throat at the level of larynx and ask him to swallow

When obtaining and interpreting anthropometric values, the nurse should recognize which of the following? MUST KNOW DEFINITION OF ANTHROPOMETRIC VALUES

the patient should be weighed on the same scale at the same time each day(weighing a patient on the same scale at the same time of day provides the most consistent data for gauging trends in the patient's weight, as shifts in fluid intake and output can alter weight significantly. The patient should also be weighed with the same amount of clothing and/ or linen each time

Which is the primary purpose for asking a patient to keep a 3 to 7 day food diary?

to assess the pattern of intake and compare with the daily reference intake

Which of the following are appropriate choices for a patient prescribed a full liquid diet?

-Plain yogurt -Custard -Pureed Vegetables -Gelatin

A patient has a deep wound on the right hip, with tunneling at the 8 o'clock position extending 5 cm. The wound is draining large amounts of serosanguineous fluid and contains 100% red beefy tissue in the wound bed. Which would be an appropriate dressing choice? 1) Alginate dressing 2) Dry gauze dressing 3) Hydrogel 4) Hydrocolloid dressing

1 1 Alginates are highly absorbent and are appropriate for wounds with moderate to large amounts of drainage. They are ideal for wounds with tunneling, as they will conform to fill the tunnel. 2 Gauze could adhere to the wound bed and cause trauma when removed. 3 A hydrogel would increase the drainage, with the potential of macerating surrounding skin. 4 Hydrocolloids have limited absorptive ability.

A patient's arterial blood gas results are pH = 7.30 PCO2 = 40 HCO3 = 19 mEq/L PO2 = 80 What would be an appropriate nursing diagnosis for the patient? 1) Impaired Gas Exchange 2) Metabolic Acidosis 3) Risk for Impaired Gas Exchange 4) Risk for Acid-Base Imbalance

1 1 An appropriate diagnosis is Impaired Gas Exchange. The arterial blood gas (ABG) results provide the defining characteristics for Impaired Gas Exchange. 2 The ABG results demonstrate metabolic acidosis; however, this is not a nursing diagnosis. 3 The patient has an actual problem; therefore, the "risk for" nursing diagnosis is incorrect. 4 There is no nursing diagnosis of Risk for Acid-Base Imbalance.

The nurse examines the site of a client's tuberculin skin test as being 5 mm induration and documents that the test is positive. Which information in the client's history did the nurse use to make this clinical determination? 1) HIV positive 2) Type 2 diabetes mellitus 3) Lives in a skilled nursing facility 4) Recent immigrant to the United States

1 1 An induration of 5 mm is considered positive for the tuberculin skin test for the client who is HIV positive. 2 An induration of 10 mm is considered positive for the tuberculin skin test for the client who has diabetes. 3 An induration of 10 mm is considered positive for the tuberculin skin test for the client who lives in a skilled nursing facility. 4 An induration of 10 mm is considered positive for the tuberculin skin test for the client who is from a country having a high incidence of TB

When applying heat or cold therapy to a wound, what should the nurse do? 1) Leave the therapy on each area no longer than 15 minutes. 2) Leave the therapy on each area no longer than 30 minutes. 3) When using heat, ensure the temperature is at least 135°F (57.2°C) before applying it. 4) When using cold, ensure the temperature is less than 32°F (0°C) before applying it.

1 1 Apply heat or cold therapies intermittently, leaving them on for no more than 15 minutes at a time in an area. This helps prevent tissue injury and also makes the therapy more effective by preventing rebound phenomenon. 2 Apply heat or cold therapies intermittently, leaving them on for no more than 15 minutes at a time in an area. This helps prevent tissue injury and also makes the therapy more effective by preventing rebound phenomenon. 3 Temperatures should be kept between 59°F and 113°F (15°C and 45°C), depending on the type of therapy chosen and what is comfortable to the patient. Temperatures colder or warmer than those recommended can damage tissue. 4 Temperatures should be kept between 59°F and 113°F (15°C and 45°C), depending on the type of therapy chosen and what is comfortable to the patient. Temperatures colder or warmer than those recommended can damage tissue

Three days after cardiac surgery, the client's leg used for harvesting veins for the bypass is warm and tender and has a 3-cm area of erythema and swelling at the distal end of the incision. The incision staples are intact without drainage and vital signs are stable. Which type of complication is this client most likely experiencing? 1) Deep vein thrombosis (DVT) 2) Dehiscence of the wound 3) Internal bleeding 4) Infection at the incisional site

1 1 DVT is a clot in the veins that are deep under the muscles of the leg. DVT can occur after surgery, after lengthy bedrest, or after trauma. Symptoms include pain, warmth, redness, and swelling of the leg. Dorsiflexion of the foot (pulling toes forward) and Pratt's sign (squeezing calf to trigger pain) have not been found to be reliable in diagnosing DVT. 2 Dehiscence is the rupture of a suture line, whereas evisceration is the protrusion of internal organs through the rupture. 3 Internal bleeding is a wound-healing complication associated with hematoma formation, pain, hypotension, and tachycardia. 4 Infection is a complication of wound healing that causes warmth, pain, inflammation of the affected area, and changes in vital signs (i.e., elevated pulse and temperature).

A patient diagnosed with hypertension is taking an angiotensin-converting enzyme (ACE) inhibitor. When planning care, which outcome would be appropriate for the patient? 1) BP will be lower than 135/85 mm Hg on all occasions. 2) BP will be normal after 2 to 3 weeks on medication. 3) Patient will not experience dizziness on rising. 4) Urine output will increase to at least 50 mL/hr

1 1 Goals must be clearly stated so that it is easy to evaluate whether they have been met. "BP . . . lower than 135/85 mm Hg . . ." is clearly stated and easily evaluated. 2 In contrast, "BP will be normal . . ." does not clearly state the desired endpoint. 3 Freedom from dizziness on rising is probably not achievable because ACE inhibitors are vasodilating agents, which may cause vessel dilation and hypotension, especially when the patient arises from a seated or lying position. Patients should be warned of this effect. 4 The expected/desired effect of the ACE inhibitor is to lower the blood pressure; the urine output is minimally relevant in determining that outcome, if at all.

A patient with a history of alcoholism is disoriented and vacillates between being calm and disruptive and loud. Vital signs are BP 138/84 mm Hg; pulse 135 beats/min, regular and strong; respiratory rate 22 breaths/min; temperature 37.1°C (98.1°F). What electrolyte imbalance might the nurse suspect this patient is experiencing? 1) Hypomagnesemia 2) Hypocalcemia 3) Hyperkalemia 4) Hypernatremia

1 1 Hypomagnesemia is a frequent consequence of alcoholism. Signs and symptoms include disorientation, mood changes, and tachycardia. 2 Hypocalcemia, a low calcium level, is associated with muscle spasms and tetany. 3 Hyperkalemia, a high potassium level, manifests as weakness, fatigue, and cardiac dysrhythmias. 4 Hypernatremia, a high sodium level, produces extreme thirst and agitation.

An older patient has been vomiting for 2 days and has been unable to eat or drink anything during that time. Current vital signs are T = 99.6°F (37.6°C) orally; P = 110 beats/min weak and thready; BP = 80/52 mm Hg. Skin and mucous membranes are dry, there is decreased skin turgor, and the patient is experiencing weakness. The most recent laboratory results are as follows: Sodium: 138 mEq/L Potassium: 3.7 mEq/L Calcium: 9.2 mg/dL Magnesium: 1.8 mg/dL Chloride: 99 mEq/L BUN: 29 mg/dL Which health problem should the nurse realize this patient is experiencing? 1) Hypovolemia 2)Hypervolemia 3) Hypernatremia 4) Hyponatremia

1 1 Hypovolemia may occur as a result of insufficient intake of fluid; bleeding; excessive loss through urine, skin, or the gastrointestinal tract; insensible losses; or loss of fluid into a third space. The first symptom of hypovolemia is thirst. Other symptoms are a rapid weak pulse, a low blood pressure (although initially the blood pressure may rise), dry skin and mucous membranes, decreased skin turgor, and decreased urine output. Temperature increases because the body is less able to cool itself through perspiration. The person with fluid volume deficit usually has elevated BUN (blood urea nitrogen) and hematocrit levels. 2 Hypervolemia involves excessive retention of sodium and water in the extracellular fluid, and the vital sign changes are opposite those of a patient with hypovolemia. 3 Hypernatremia is not applicable because the patient's sodium level is within normal range. 4 Hyponatremia is not applicable because the patient's sodium level is within normal range.

When a patient has metabolic acidosis, which body system influences the acid-base imbalance to produce the compensatory changes in the arterial blood gases? 1) Respiratory system 2) Renal system 3) Vascular system 4) Neurological system

1 1 In a metabolic problem, the respiratory system compensates. 2 In a respiratory problem, the renal system must compensate. 3 The vascular system does not participate in acid-base balance. 4 The neurological system does not participate in acid-base balance.

Three days after abdominal surgery, the nurse notes 4-cm periwound erythema and swelling at the distal end of a client's incision. The area is tender and warm to the touch. Staples are intact along the incision, and there is no obvious drainage. Heart rate is 96 beats/min and oral temperature is 100.8°F (38.2°C). What kind of complication should the nurse suspect this client is experiencing? 1) Infection at the incisional site 2) Dehiscence of the wound 3) Hematoma under the skin 4) Formation of granulation tissue

1 1 Infection is a complication of wound healing that causes warmth, pain, inflammation of the affected area, and changes in vital signs (i.e., elevated pulse and temperature). 2 Dehiscence is the rupture of a suture line. 3 A hematoma is a collection of blood that forms under the skin. It is usually tender or painful to the touch and is usually swollen. 4 Granulation tissue is new connective tissue and tiny blood vessels that form on the surfaces of a wound during the healing process. It is beefy red in appearance but would not be warm or tender to the touch.

Which body fluid lies in the spaces between the body cells? 1) Interstitial 2) Intracellular 3) Intravascular 4) Transcellular

1 1 Interstitial fluid lies in the spaces between the body cells. 2 Intracellular fluid is contained within the cells. 3 Intravascular fluid is the plasma within the blood. 4 Transcellular fluid includes specialized fluids, such as cerebrospinal, pleural, peritoneal, and synovial, and digestive juices.

The nurse identifies that a client is at risk for developing cardiovascular system problems. Which information did the nurse use to make this clinical determination? 1) Works as a computer operator 2) Stopped smoking decades ago 3) Drinks coffee without cream or sugar 4) Ingests fresh fruit and vegetables daily

1 1 Lack of exercise and a sedentary lifestyle are risk factors for the development of cardiovascular disease. 2 Smoking cessation is a positive action to reduce the risk of developing cardiovascular disease. 3 Caffeine intake is not identified as a risk factor for cardiovascular disease. 4 Ingesting nutritious foods reduces the risk of developing cardiovascular disease.

A patient with quadriplegia has an ischial wound that extends through the epidermis into the dermis. How should the nurse document the depth of this wound? 1) Partial-thickness wound 2) Penetrating wound 3) Superficial wound 4) Full-thickness wound

1 1 Partial-thickness wounds extend through the epidermis into the dermis. 2 Penetrating is a descriptor sometimes added to indicate that the wound includes internal organs. 3 Superficial wounds involve only the epidermal layer of skin. 4 Full-thickness wounds extend into the subcutaneous tissue and beyond.

The nurse notes that a client has a prescription for a peak expiratory flow meter. For which health problem should the nurse prepare teaching for this client? 1) Asthma 2) Pneumonia 3) Emphysema 4) Pulmonary edema

1 1 Peak expiratory flow rate (PEFR) measures the amount of air that can be exhaled with forcible effort. Patients with asthma use PEFR monitoring to detect subtle changes in their condition, often before symptoms occur. 2 PEFR is not used for clients with pneumonia. 3 PEFR is not used for clients with emphysema. 4 PEFR is not used for clients with pulmonary edema.

What is the rationale for wrapping petroleum gauze around a chest tube insertion site? 1) Prevents air from leaking around the site 2) Prevents infection at the insertion site 3) Absorbs drainage from the insertion site 4) Protects the tube from becoming dislodged

1 1 Petroleum gauze creates a seal around the insertion site. Collapse of the lung can occur if there is a leak around the insertion site that causes loss of negative pressure within the system. Air leaks are one common cause of loss of negative pressure. 2 Petroleum gauze is not used to reduce infection. 3 Petroleum gauze is not used to absorb drainage from the insertion site. 4 Petroleum gauze does not protect the tube from becoming dislodged.

Pressure ulcers are directly caused by which condition at the site? 1) Compromised blood flow 2) Edema 3) Shearing forces 4) Inadequate venous return

1 1 Pressure ulcers are caused by unrelieved pressure that compromises blood flow to an area, resulting in ischemia (inadequate blood supply) in the underlying tissue. 2 Edema leads to compromised skin and tissue integrity, which is more prone to pressure injury. 3 Friction and shear are extrinsic factors affecting skin integrity, which increases the risk of a client developing a pressure ulcer but is not the direct cause. 4 Inadequate arterial blood flow to an area due to pressure causes the development of a pressure ulcer.

A client experiences leg pain and edema of both ankles. Which laboratory test should the nurse anticipate being prescribed for this client? 1) C reactive protein (CRP) 2) Coagulation studies 3) Complete blood count 4) Complete metabolism panel

1 1 The CRP appears to be the most reliable marker for arterial inflammation currently available. This would be appropriate for the client with leg pain and ankle edema. 2 There is no reason for coagulation studies to be completed. 3 A complete blood count might be indicated to measure oxygenation of the blood; however, it would not directly help determine the presence of inflammation that could be causing the leg pain and ankle edema. 4 A complete metabolism panel is done on clients upon hospitalization but these values would not be used to identify the reason for leg pain and ankle edema.

Why is the information obtained from a swab culture of a wound limited? 1) A positive culture does not necessarily indicate infection, because chronic wounds are often colonized with bacteria. 2) A negative culture may not indicate infection, because chronic wounds are often colonized with bacteria. 3) Most wound infections are viral, so the swab culture would not be indicative of a wound infection. 4) A swab culture result does not include bacterial sensitivity information necessary to provide treatment.

1 1 The information obtained from a swab culture is limited because a positive culture may not indicate infection. 2 Chronic wounds are often colonized with bacteria, but this does not require antibiotic treatment. 3 Most wound infections are bacterial. 4 A needle aspiration of the wound would provide more definitive information about whether the wound is infected or not and can be performed by a registered nurse. However, the most accurate wound information is obtained by tissue biopsy performed by a specially trained provider.

A client recovering from an acute myocardial infarction is experiencing tachycardia, palpitations, anxiety, jugular vein distention, and fatigue. Which nursing diagnoses is most appropriate for this client? 1) Decreased Cardiac Output 2) Ineffective Peripheral Tissue Perfusion 3) Impaired Cardiac Contractility 4) Activity Intolerance

1 1 The patient's symptoms reflect altered cardiac preload, a component of cardiac output. Acute myocardial infarction is often associated with decreased cardiac output as a result of altered cardiac pumping ability. 2 Ineffective Peripheral Tissue Perfusion cannot be determined from the client's symptoms. 3 Impaired Cardiac Contractility is not a NANDA-I nursing diagnosis. 4 Activity Intolerance cannot be determined from the client's symptoms.

A client is prescribed an ACE inhibitor. What should the nurse explain as the reason for this medication? 1) Dilate vessels to ease the workload of the heart 2) Increase venous return to raise the blood pressure 3) Decrease the heart rate and reduce heart contractility 4) Increase removal of sodium and water from the body

1 1 Vasodilators cause vessel dilation, which eases the work of the heart. Vasodilating agents include ACE inhibitors. 2 Vasodilators decrease blood pressure. 3 Beta-adrenergic agents decrease the heart rate and reduce myocardial contractility. 4 Diuretics remove sodium and water from the body.

Which statement describes the difference between dehiscence and evisceration? 1) With dehiscence, there is a separation of one or more layers of wound tissue; evisceration involves the protrusion of internal viscera from the incision site. 2) Dehiscence is an urgent complication that requires surgery as soon as possible; evisceration is not as urgent. 3) Dehiscence involves the protrusion of internal viscera from the incision site; with evisceration, there is a separation of one or more layers of wound tissue. 4) Dehiscence involves rupture of subcutaneous tissue; evisceration involves damage to dermal tissue.

1 1 With dehiscence there is a separation of one or more layers of wound tissue, whereas evisceration involves the protrusion of internal viscera from the incision site. 2 Evisceration is an urgent complication usually requiring immediate surgical intervention. 3 With dehiscence there is a separation of one or more layers of wound tissue, whereas evisceration involves the protrusion of internal viscera from the incision site. 4 With dehiscence there is a separation of one or more layers of wound tissue, whereas evisceration involves the protrusion of internal viscera from the incision site.

What is the primary goal that the nurse should establish for a patient with an open wound? 1) The wound will remain free of infection throughout the healing process. 2) Client completes antibiotic treatment as ordered. 3) The wound will remain free of scar tissue at healing. 4) Client increases caloric intake throughout the healing process.

1 1 Wounds healing by secondary intention are more prone to infection; therefore, the primary goal is to prevent infection. 2 Antibiotics may not be necessary. 3 The nurse can expect the formation of scar tissue in this particular situation. 4 There is no evidence presented that the patient needs to increase caloric intake.

The nurse assesses a patients abdomen 4 days after abdominal surgery and notes that bowel sounds are absent. This finding most likely suggest which postoperative complication? 1) Paralytic ileus 2) Small bowel obstruction 3) Diarrhea 4) Constipation

1 Absent bowel sounds on the fourth postoperative day suggests paralytic ileus, a complication associated with abdominal surgery. A small bowel obstruction and diarrhea produce hyperactive bowel sounds. Constipation might be associated with hypoactive bowel sounds

A patient is diagnosed with an intestinal infection after traveling abroad. The nurse should encourage the intake of which food to promote healing? 1) Yogurt 2) Pasta 3) Oatmeal 4) Broccoli

1 Although the patient may have diarrhea, the goal is not to stop the diarrhea, but to eliminate the pathogens from the digestive tract. The active bacteria in yogurt stimulate peristalsis and promote healing of intestinal infections. Pasta is a low-fiber food that slows peristalsis. It does not promote healing of intestinal infections. Oatmeal stimulates peristalsis, but it does not promote healing of intestinal infections. Broccoli stimulates gas production; it is ineffective against intestinal infections

A patient with severe hemorrhoids is incontinent of liquid stool. Which of the following interventions is contraindicated? 1) Apply an indwelling fecal drainage device. 2) Apply an external fecal collection device. 3) Place an incontinence garment on the patient. 4) Place a waterproof pad under the patients buttocks.

1 An indwelling fecal drainage device is contraindicated for children; for more than 30 consecutive days of use; and for patients who have severe hemorrhoids, recent bowel, rectal, or anal surgery or injury; rectal or anal tumors; or stricture or stenosis. External devices are not typically used for patients who are ambulatory, agitated, or active in bed because the device may be dislodged, causing skin breakdown. External devices cannot be used effectively when the patient has Impaired Skin Integrity because they will not seal tightly. Absorbent products are not contraindicated for this patient unless Impaired Skin Integrity occurs. Even with absorbent products or an external collection device, the nurse should place a waterproof pad under the patient to protect the bed linens.

Which food provides the body with no usable glucose? 1) Wheat germ 2) Apple 3) White bread 4) White rice

1 Dietary fiber, such as wheat germ, contains no usable glucose. Apples, white bread, and white rice all contain carbohydrates, which provide usable glucose

A patient who was prescribed furosemide (Lasix) is deficient in potassium. Which of the following is an appropriate goal for this patient? The patient will increase his consumption of: 1) Bananas, peaches, molasses, and potatoes. 2) Eggs, baking soda, and baking powder. 3) Wheat bran, chocolate, eggs, and sardines. 4) Egg yolks, nuts, and sardines.

1 Foods rich in potassium include bananas, peaches, molasses, meats, avocados, milk, shellfish, dates, figs, and potatoes. Eggs, baking soda, and baking powder have a high sodium content. Dairy products, beef, pork, beans, sardines, eggs, chicken, wheat bran, and chocolate are rich in phosphorus. Egg yolks, nuts, sardines, dairy products, broccoli, and legumes are rich in calcium

1. What is the most significant change in kidney function that occurs with aging? 1) Decreased glomerular filtration rate 2) Proliferation of micro blood vessels to renal cortex 3) Formation of urate crsystals 4) Increased renal mass

1 Glomerular filtration rate is the amount of filtrate formed by the kidneys in 1 minute. Renal blood flow progressively decreases with aging primarily because of reduced blood supply through the micro blood vessels of the kidney. A decrease in glomerular filtration is the most important functional deficit caused by aging. Urate crystals are somewhat common in the newborn period. They might indicate that the infant is dehydrated. In older people, they result from too much uric acid in the blood, although this is not related to aging. Renal mass (weight) decreases over time, starting around age 30 to 40.

The nurse notices that a patient has spoon-shaped, brittle nails. This suggests that the patient is experiencing Imbalanced Nutrition: Less Than Body Requirements related to deficiency of which of the following nutrients? 1) Iron 2) Vitamin A 3) Protein 4) Vitamin C

1 Patients with iron deficiency may have spoon-shaped, brittle nails. Other abnormal nail findings include dull nails with transverse ridge (protein deficiency); pale, poor blanching, or mottled nails (vitamin A or C deficiency); splinter hemorrhages (vitamin C deficiency); and bruising or bleeding beneath nails (protein or caloric deficiency).

A patient is brought to the emergency department experiencing leg cramps. He is irritable, his temperature is elevated, and his mucous membranes are dry. Based on these findings, the patient most likely has excess levels of which mineral? 1) Sodium 2) Potassium 3) Phosphorus 4) Magnesium

1 Signs and symptoms associated with sodium excess include thirst, fever, dry and sticky tongue and mucous membranes, restlessness, irritability, and seizures. Findings associated with potassium excess include cardiac arrhythmias, weakness, abdominal cramps, diarrhea, anxiety, and paresthesia. Phosphorus excess leads to tetany and seizures. Magnesium excess causes weakness, nausea, and malaise

The nurse is providing nutrition counseling for a patient planning pregnancy. The nurse should emphasize the importance of consuming which nutrient to prevent neural tube defects? 1) Folic acid 2) Calcium 3) Protein 4) Vitamin D

1 The nurse should emphasize the importance of consuming folic acid even before conception to prevent neural tube defects from developing. Calcium and protein needs also increase during pregnancy; however, their consumption does not prevent neural tube defects. Vitamin D consumption does not prevent neural tube defects.

A mother of a school-age child seeks healthcare because her child has had diarrhea after being ill with a viral infection. The patient states that after vomiting for 24 hours, his appetite has returned. Which recommendation should the nurse make to this mother? 1) Consume a diet consisting of bananas, white rice, applesauce, and toast. 2) Drink large quantities of water regularly to prevent dehydration. 3) Take loperamide [an antidiarrheal] as needed to control diarrhea. 4) Increase the consumption of raw fruits and vegetables.

1 The nurse should encourage the patient with diarrhea who has an appetite to consume a diet that consists of bananas, white rice, applesauce, and toast. These foods are easy to digest, provide calories for energy, and help provide a source of calcium. The patient should sip liquids frequently to prevent dehydration; large quantities might worsen diarrhea. Medication, such as loperamide (Imodium), is usually reserved for chronic diarrhea. Raw fruits and vegetables may worsen diarrhea.

The nurse is instructing a patient about performing home testing for fecal occult blood. The nurse can conclude that learning occurs if the patient says, For 3 days prior to testing, I should avoid eating 1) Beef. 2) Milk. 3) Eggs. 4) Oatmeal.

1 The nurse should instruct the patient to avoid red meat, chicken, fish, horseradish, and certain raw fruits and vegetables for 3 days prior to fecal occult blood testing.

A mother brings her 4-month-old infant for a well-baby checkup. The mother tells the nurse that she would like to start bottle feeding her baby because she cannot keep up with the demands of breastfeeding since returning to work. Which response by the nurse is appropriate? 1)Make sure you give your baby an iron-fortified formula to supplement any stored breast milk you have. 2) You really need to continue breastfeeding your baby. 3) Give your baby formula until he is 6 months old; then you can introduce whole milk. 4) Your baby weighs 14 pounds, so he will require about 36 ounces of formula a day.

1 The nurse should not make the mother feel guilty about her decision to stop breastfeeding. Instead, she should provide the mother with instruction about bottle feeding. She can give it to supplement any stored breast milk she might have in supply. She should emphasize the importance of giving the baby iron-fortified formula because fetal iron stores become depleted by 4 to 6 months of age. Infants younger than 1 year of age should not receive regular cows milk because it may place a strain on the immature kidneys. Because the baby weighs 14 pounds, he will require about 21 ounces of formula a day (not 36 ounces), based on the nutritional recommendations that infants require 80 to 100 ml of formula or breast milk per kilogram of body weight per day

Which portion of a nutritional assessment must the registered nurse complete? 1) Analyzing the data 2) Obtaining intake and output 3) Weighing the patient 4) Obtaining the history

1 The registered nurse should review and interpret the data collected as part of a nutritional assessment. The registered nurse can delegate height, weight, and intake and output to nursing assistive personnel. History taking can be safely delegated to the licensed practical nurse.

A 52-year-old man has a triceps skinfold thickness of 18 mm, and his weight exceeds the ideal body weight for his height by 23%. Which nursing diagnosis should the nurse identify for this patient? 1) Imbalanced Nutrition: More Than Body Requirements 2) Risk for Imbalanced Nutrition: More Than Body Requirements 3) Imbalanced Nutrition: Less Than Body Requirements 4) Readiness for Enhanced Nutrition

1 This patient has defining characteristics for the nursing diagnosis Imbalanced Nutrition: More Than Body Requirements: triceps skinfold thickness more than 15 mm in men and weight that is 20% over ideal for height and frame. The patient does not have defining characteristics for the other nursing diagnoses

Which action should the nurse take to assess a 2 year-old child for pinworms? 1) Press clear cellophane tape against the anal opening to obtain a specimen. 2) Collect a freshly passed stool from a diaper using a wooden spatula. 3) Place a smear of stool on a slide and add two drops of reagent. 4) Prepare the patient for a flat plate (x-ray) of the abdomen.

1 To assess for pinworms, the nurse should press cellophane tape against the childs anal opening during the night or as soon as he awakens. Remove the tape immediately, and place it on a slide. Perineal swabs may also be necessary for microscopic study. Collecting a fresh stool specimen from a diaper describes the method for an infant or toddler. Placing a smear of stool on a slide and adding a reagent describes fecal occult blood testing. An abdominal flat plate is not a method of assessing for pinworms.

The nurse identifies the nursing diagnosis Urinary Incontinence (Total) in an older adult patient admitted after a stroke. Urinary Incontinence places the patient at risk for which complication? 1) Skin breakdown 2) Urinary tract infection 3) Bowel incontinence 4) Renal calculi

1 Urine contains ammonia, which may cause excoriation with prolonged contact with the skin. Bowel incontinence, not urinary incontinence, increases the patient's risk for urinary tract infection. Immobility and high consumption of calcium-containing foods increase the risk for renal calculi.

The nurse measures the urine output of a patient who requires a bedpan to void. Which action should the nurse take first? Put on gloves and: 1) Have the patient void directly into the bedpan. 2) Pour the urine into a graduated container. 3) Read the volume with the bedpan on a flat surface at eye level. 4) Observe color and clarity of the urine in the bedpan.

1 - First, the nurse should put on gloves and have the patient void directly into the bedpan. Next, she should pour the urine into a graduated container, place the measuring device on a flat surface, and read the amount at eye level. She should observe the urine for color, clarity, and odor. Then, if no specimen is required, she should discard the urine in the toilet and clean the container and bedpan. Finally, she should record the amount of urine voided on the patients intake and output record.

The surgeon orders hourly urine output measurement for a patient after abdominal surgery. The patients urine output has been greater than 60 ml/hour for the past 2 hours. Suddenly the patients urine output drops to almost nothing. What should the nurse do first? 1) Irrigate the catheter with 30 ml of sterile solution. 2) Replace the patients indwelling urinary catheter. 3) Infuse 500 ml of normal saline solution IV over 1 hour. 4) Notify the surgeon immediately.

1 If the patients urinary output suddenly ceases, the nurse should irrigate the urinary catheter to assess whether the catheter is blocked. If no blockage is detected, the nurse should notify the surgeon. The surgeon may request that the catheter be changed if irrigation does not help or if the tubing is not kinked. However, the nurse should not change a catheter in the immediate postoperative period without consulting with the surgeon. The surgeon may prescribe an IV fluid bolus if the patient is suspected to have a deficient fluid volume.

The student nurse asks the provider if she will prescribe an indwelling urinary catheter for a hospitalized patient who is incontinent. The provider explains that catheters should be utilized only when absolutely necessary because: 1) They are the leading cause of nosocomial infection. 2) They are too expensive for routine use. 3) They contain latex, increasing the risk for allergies. 4) Insertion is painful for most patients.

1 Indwelling urinary catheters should not be routinely used for hospitalized patients with incontinence because they are the leading cause of healthcare-acquired infection (nosocomial). The cost of an indwelling urinary catheter should not deter its use if necessary. Latex-free catheters are available for patients with or at risk for latex allergy. Insertion may be somewhat uncomfortable, but it should not be painful.

A mother tells the nurse at an annual well child checkup that her 6-year-old son occasionally wets himself. Which response by the nurse is appropriate? 1) Explain that occasional wetting is normal in children of this age 2) Tell the mother to restrict her childs activities to avoid wetting 3) Suggest time-out to reinforce the importance of staying dry 4) Inform the mother that medication is commonly used to control wetting

1 The nurse should explain that occasional wetting is normal in children during the early school years. The mother should handle the situation calmly and avoid punishing the child. Medications are occasionally prescribed for nocturnal enuresis when the child is older and not sleeping at home, but not for occasional daytime wetting.

Which action should the nurse take when beginning bladder training using scheduled voiding? 1) Offer the patient a bedpan every 2 hours while she is awake. 2) Increase the voiding interval by 30 to 60 minutes each week. 3) Frequently ask the patient if she has the urge to void. 4) Increase the frequency between voiding even if urine leakage occurs.

1 The nurse should offer the patient the bedpan or assist the patient to the bathroom every 2 hours while she is awake. You would encourage the patient to get up once during the night to void but awakening the patient every 2 hours would lead to fatigue. If the patient adheres to the schedule, the voiding interval should be increased by 15 to 30 minutes each week. The nurse should not frequently ask the patient about the urge to void.

A nurse is planning on obtaining a urinary specimen from a patient's closed urinary system. Identify the sequence of steps the nurse should take. -Transport the specimen to the lab -Insert 10ml syringe and needle into port -Withdraw 5ml of urine -Wipe port with alcohol swab -Transfer urine to a sterile specimen container

1) Wipe port with alcohol swap 2) Insert a 10 mL syringe and needle into port. 3)Withdraw 5 mL of urine 4) Transfer the urine to a sterile specimen container 5) Transport the specimen to the lab

To promote wound healing, the nurse is teaching a patient about choosing foods containing protein. The nurse will evaluate that learning has occurred if the patient recognizes which food(s) as an incomplete protein that should be consumed with a complementary protein? Choose all that apply. 1) Whole grain bread 2) Peanut butter 3) Chicken 4) Eggs

1,2 Incomplete protein foods do not provide all of the essential amino acids necessary for protein synthesis. Therefore, the nurse should inform the patient that whole grain bread and peanut butter should be consumed with a complementary protein. For example, they could be eaten together as a peanut butter sandwich.

The nurse is teaching a patient about the importance of reducing saturated fats in his diet. The nurse will recognize that learning has occurred if, upon questioning, the patient replies that he should read product labels to eliminate the intake of which saturated fat(s)? Choose all that apply. 1) Palm oil 2) Coconut oil 3) Canola oil 4) Peanut oil

1,2 Palm and coconut oils are sources of saturated fat that are contained in many processed foods. The patient should be encouraged to read product labels to eliminate them from his diet. Olive, canola, and peanut oils are unsaturated fats and should be substituted for saturated fats in the diet.

Which factor(s) place(s) the patient at risk for constipation? SATA: 1) Sedentary lifestyle 2) High-dose calcium therapy 3) Lactose intolerance 4) Consuming spicy foods

1,2 Physical activity stimulates peristalsis and bowel elimination. Therefore, those with a sedentary lifestyle commonly experience constipation. High-dose calcium therapy also predisposes a patient to constipation. Lactose intolerance and consuming spicy foods are associated with a nursing diagnosis of Diarrhea, not Constipation

An older client reports fatigue when walking upstairs and when performing normal household activities. Which age-related change in the cardiovascular system might the client be experiencing? Select all that apply.. 1) Cardiac contractile strength is reduced. 2) Heart valves become more rigid. 3) Peripheral vessels lose elasticity. 4) Heart responds to increased oxygen demands. 5) There is reduced tolerance to exercise.

1,2,3,5 1. Cardiac efficiency gradually declines as the heart muscle loses contractile strength. 2. Heart valves become thicker and more rigid with aging. 3. The peripheral vessels become less elastic, which creates more resistance to ejection of blood from the heart. 4. As a result of these changes, the heart becomes less able to respond to increased oxygen demands, and it needs longer recovery times after responding. 5. Older adults have lower exercise tolerance, need more rest after exercise, and are more prone to orthostatic hypotension.

In a healthy adult, which regulates body fluids? Select all that apply. 1) Hormone levels 2) Fluid intake 3) Oxygen saturation 4) Kidney function 5) Neurological function

1,2,4 1. Specific hormones (e.g., ADH, aldosterone) cause the kidneys to regulate the body's fluid and electrolyte balance. 2. A balance between fluid intake and output is essential to maintain homeostasis. Excesses or deficits of intake can lead to severe disorders. 3. Oxygen saturation does not regulate fluids. It measures the saturation of oxygen on hemoglobin and is influenced by the partial pressure of oxygen, alveolar-arterial gradient lung disease, and the amount and type of hemoglobin (such as sickle cell anemia). 4. The kidneys are the principal regulator of fluid and electrolyte balance and are the primary source of fluid output. 5. The neurological system does not have an active role in the regulation of body fluids.

Which outcome is related to Decreased Cardiac Output? Select all that apply. 1) No dyspnea or shortness of breath with exertion 2) Normal skin color 3) Respiratory rate less than 16 breaths/min 4) Brisk capillary refill 5) Urine output > 50 mL/hr

1,2,4,5 1. Absence of dyspnea or shortness of breath with exertion indicates a well-functioning cardiac output. 2. Normal skin color indicates an adequate cardiac output. 3. Respiratory rate of less than 16 breaths/min is hypoventilation and can lead to poor oxygenation and tissue acidosis. 4. Brisk capillary refill indicates an adequate cardiac output. 5. A urine output > 50 mL/hr indicates adequate renal perfusion and adequate cardiac output.

A patient with COPD has a pulse oximetry reading of 97%. What other finding would indicate adequate tissue and organ oxygenation? Select all that apply. 1) Normal urine output 2) Strong peripheral pulses 3) Clear breath sounds bilaterally 4) Normal muscle strength 5) Orientation

1,2,4,5 1. Impaired tissue oxygenation to the kidneys would result in abnormal kidney function (e.g., poor urine output). 2. Good peripheral circulation is characterized by strong peripheral pulses. 3. Adequacy of tissue oxygenation cannot be determined by assessing pulmonary ventilation alone; circulation must also be assessed. 4. Hypoxic limb tissue would result in abnormal muscle functioning (e.g., muscle weakness and pain with exercise). 5. If a patient is oriented, adequate oxygen is reaching the cerebrum.

A client had a myocardial infarction that damaged the sinoatrial (SA) node. For which potential complication should the nurse assess this client? Select all that apply. 1) Decreased heart rate 2) Increased heart rate 3) Decreased cardiac output 4) Decreased strength of ventricular contractions 5) Increased cardiac output

1,3 1. Normally, the SA node is the primary pacemaker for the heart and initiates a rate of 60 to 100 beats per minute. If the SA node fails, the atrioventricular node can take over as the pacemaker, but it generally triggers a slower heart rate. 2. The heart rate will be slower. 3. Cardiac output will decrease as a result of the decrease in heart rate. 4. Damage to the SA node interferes with the electrical activity of the heart but does not directly affect the pumping action of the heart. 5. Cardiac output will decrease as a result of the decrease in heart rate.

The nurse needs to relocate an intravenous site in a patient with an infiltrated IV in the right forearm. In which site is it appropriate for the nurse to insert the new IV line? Select all that apply. 1) Left hand 2) Right wrist 3) Right antecubital area 4) Right saphenous vein 5) Left jugular vein

1,3 1. The opposite extremity (e.g., left hand) may also be used. 2. The IV should be started above the site of the infiltration. 3. When restarting an IV line after an infiltration, you must restart above the site of infiltration. As a result, the right antecubital area is correct. 4. The right saphenous vein is incorrect because that vein is located in the leg. The leg should be used as a last resort for an IV site. The primary care provider should be notified if a leg is being considered as an IV site. 5. The nurse would not use the jugular vein for an IV infusion.

Which instruction(s) should the nurse give to the patient complaining of constipation? Choose all that apply. 1) Drink at least 8 glasses of water or fluid per day. 2) Include a minimum of 4 servings of meat per day. 3) Consume a high-fiber diet. 4) Exercise as you feel necessary.

1,3 To prevent constipation, the nurse should instruct the patient to consume a high-fiber diet; drink at least 8 glasses of water or fluid per day; exercise regularly; and eat meals on a regular schedule

When assisting with bedside central venous catheter (CVC) placement, which nursing intervention is appropriate? Select all that apply. 1) Apply sterile gloves and mask (and possibly gown). 2) Scrub the insertion site with antibacterial soap for 1 min. 3) Verify that informed consent has been obtained. 4) Place the patient in low Fowler's position. 5) Prepare an infusion bag with dextrose 10% and water.

1,3 1. Maximum barrier sterile technique is used for CVC insertion (sterile gloves, mask, and gown), although some agency policies do not include sterile gown for the nurse. 2. The scrub is not done with antibacterial soap. The scrub is done with chlorhexidine-alcohol solution or, alternatively, first with 70% alcohol and then with povidone detergent. 3. This is an invasive procedure, so informed consent is required. The nurse should confirm that this has been obtained. 4. The patient is placed in Trendelenburg position with a rolled towel between the shoulders for best site access. 5. Dextrose 10% and water is not the standard infusion solution for a central venous catheter.

What is the mechanism that controls acid-base balance? Select all that apply. 1) Respiratory mechanisms 2) Active transport mechanisms 3) Renal mechanisms 4) Buffer systems 5) Peripheral vascular system

1,3,4 1. The lungs (respiratory mechanisms) control the carbonic acid supply via carbon dioxide. Conditions that cause retention of carbon dioxide, such as chronic obstructive pulmonary disease, lower the pH, whereas tachypneic conditions, such as hyperventilation syndrome, "blow off" carbon dioxide and increase the pH. 2. Active transport involves the movement of fluids and electrolytes in the body. 3. The kidneys (renal mechanisms) regulate the concentration of plasma bicarbonate. By reabsorbing or excreting bicarbonate, the kidneys affect acid-base balance. 4. Buffer systems prevent wide swings in pH by absorbing or releasing free hydrogen ions. 5. The peripheral vascular system does not control acid-base balance.

Which is an example of a nonselective mechanical débridement method? Select all that apply. 1) Wet-to-dry dressings 2) Sharp débridement 3) Whirlpool 4) Pulsed lavage 5) Enzymes

1,3,4 1. West-to-dry dressings are nonselective forms of mechanical débridement because health tissue and devitalized tissue can be removed. 2. Sharp débridement is a selective form of débridement. With sharp débridement, only devitalized tissue is removed. 3. Whirlpool is a nonselective form of mechanical débridement because health tissue and devitalized tissue can be removed. 4. Pulsed lavage is a nonselective form of mechanical débridement because health tissue and devitalized tissue can be removed. 5. Enzymatic débridement uses proteolytic agents to break down necrotic tissue without affecting viable tissue in the wound.

The nurse is counseling a 17-year-old girl on smoking cessation. What should the nurse include when teaching this client? Select all that apply. 1) "Keep healthy snacks or gum available to chew instead of smoking a cigarette." 2) "Don't tell your friends and family you are trying to quit, until you feel confident that you'll be successful." 3) "Plan a time to quit when you will not have many other demands or stressors in your life." 4) "Reward yourself with an activity you enjoy when you quit smoking." 5) "Spend time with friends who do not smoke."

1,3,4,5 1. Having something to chew (e.g., carrot sticks, gum, nuts, or seeds) can distract from the desire to smoke a cigarette. 2. People who are trying to quit smoking often are more successful when they are accountable to other people who are encouraging and supportive. 3. Setting a date to stop smoking and choosing a time of low stress are two strategies that help people be more successful with smoking cessation. 4. Self-reward for meeting goals is a form of positive reinforcement. 5. Identify friends who do not smoke and plan to spend time with them.

Which factor influences normal lung volumes and capacities? Select all that apply. 1) Age 2) Race 3) Body size 4) Activity level 5) Gender

1,3,4,5 1. Normal lung volumes and capacities vary with age. 2. Normal lung volumes and capacities are not impacted by race. 3. Normal lung volumes and capacities vary with body size. 4. Normal lung volumes and capacities vary with exercise level. 5. Men have greater lung volume and capacity for ventilation.

A patient who has been immobile since sustaining injuries in a motor vehicle accident complains of constipation. The nurse encourages him to consume 8 to 10 eight-ounce glasses of fluid daily. Which fluid(s) should the patient avoid because of the diuretic effect? Choose all that apply. 1) Cranberry juice 2) Water 3) Coffee 4) Ginger ale 5) Tea

1,3,5 Coffee, tea, and caffeine-containing sodas should be avoided because caffeine promotes diuresis, placing the patient at further risk for constipation. Water is the preferred fluid; however, fruit juices and decaffeinated sodas are also acceptable.

A client has a chest drainage system. What should the nurse include when teaching the client about this system? Select all that apply. 1) Perform frequent coughing and deep-breathing exercises. 2) Sit up in a chair but do not walk while the drainage system is in place. 3) Get out of bed without assistance as much as possible. 4) Immediately notify the nurse if experiencing increased shortness of breath. 5) Make sure the collection device is above the level of the chest tube insertion site.

1,4 1. Patients should regularly perform coughing and deep-breathing exercises to promote lung reexpansion. 2. To promote lung reexpansion, the nurse should encourage the patient to be as active as the condition permits, rather than telling the patient not to walk. 3. Chest drainage systems are bulky, but patients with disposable systems can still get out of bed and ambulate. However, the patient will need assistance from one or two staff members to protect and monitor the system and to monitor responses to activity; the patient should not get out of bed without assistance. 4. If a patient with a chest drainage system becomes acutely short of breath, the patient should immediately notify the nurse so the nurse can check for occlusion of the system, which can result in a tension pneumothorax. 5. The chest drainage collection device should always be below the level of the chest tube insertion site.

Why is an accurate description of the location of a wound important? Select all that apply. 1) Influences the rate of healing 2) Determines the appropriate treatment choice 3) Will affect the frequency of dressing changes 4) Affects patient movement and mobility 5) Used to determine the amount of scarring

1,4 1. Wounds in highly vascular areas heal more rapidly than wounds in less vascular regions. 2. Treatment choices will be dependent on the condition of the wound, not the location. 3. Frequency of dressing changes will be dependent on the condition of the wound, not the location. 4. Wounds that can be stabilized also heal more readily than those in areas of stress. 5. The amount of scarring depends on the healing approach used.

For a patient with Risk for Imbalanced Nutrition: Less Than Body Requirements related to Impaired Swallowing, which nursing interventions are appropriate? Choose all that apply. 1) Check inside the mouth for pocketing of food after eating. 2) Provide a full liquid diet that is easy to swallow. 3) Remind the patient to raise the chin slightly to prepare for swallowing. 4) Keep the head of the bed elevated for 30 to 45 minutes after feeding

1,4 The nurse should check for pocketing of food that the patient has not been able to swallow, and should keep the head of the bed elevated for 30 to 45 minutes after feeding. Liquids should be avoided unless thickeners are added. The patient should flex the head forward (tuck the chin) in preparation for swallowing.

Which statement is accurate about nasotracheal suctioning? Select all that apply. 1) Apply suction for no longer than 10 to 15 seconds during a single pass. 2) Apply suction while inserting and removing the catheter. 3) Reapply oxygen between suctioning passes for ventilator patients. 4) Gently rotate the suction catheter as you remove it. 5) Allow intervals of at least 30 seconds between suctioning.

1,4,5 1. Limiting suctioning to 10 seconds or less and reapplying oxygen between suctioning passes prevent hypoxia. 2. Suction should be applied only while withdrawing the catheter. 3. Endotracheal suctioning is used when the patient is being mechanically ventilated, and most ventilator patients have in-line suctioning, so there is no need to reapply oxygen. 4. Suction should be applied using a continuing rotating motion to prevent trauma to the airway. 5. Each pass of suctioning should be separated by 30 seconds.

A patient with a colostomy complains to the nurse, I am having really bad odors coming from my pouch. To help control odor, which foods should the nurse advise him to consume? 1) White rice and toast 2) Tomatoes and dried fruit 3) Asparagus and melons 4) Yogurt and parsley

1? 4? idk sorry bitches

A patient has finished a 16-oz container of orange juice. The intake and output sheet documents fluid in milliliters. Which of the following should the nurse document as intake? CONVERSION FROM OZ TO mL

1oz=30mL 480 mL

A patient with trigeminal neuralgia is prescribed a mechanical soft diet. This diet places the patient at risk for which complication? 1) Dehydration 2) Constipation 3) Hyperglycemia 4) Diarrhea

2

What is a common characteristic of aging skin? 1) Increased permeability to moisture 2) Diminished sweat gland activity 3) Reduced oxygen-free radicals 4) Overproduction of elastin

2 1 An infant's skin is thinner and more permeable to moisture in the environment. 2 Sweat gland activity is diminished. Aging skin tends to be drier. 3 Skin aging also occurs with exposure to oxygen-free radicals that are waste products from chemical reactions in the body as well as with exposure to certain food and environmental sources. 4 The skin's connective tissue, collagen, and elastin are reduced, which means the skin loses firmness and so wrinkles.

A client has a heart rate of 112 beats per minute. Which type of dysrhythmia should the nurse suspect this client is experiencing? 1) Brady 2) Tachy 3) Junctional 4) Supraventricular

2 1 Brady dysrhythmias have rates < 60 beats/min. 2 Tachy dysrhythmias have rates > 100 beats/min. 3 A junctional dysrhythmia occurs within the atrioventricular node. 4 A supraventricular dysrhythmia occurs above the ventricles. More information is needed to determine whether the client is experiencing a supraventricular dysrhythmia.

What impact does hypertensive medication have on skin integrity and wound healing? 1) Causes cellular toxicity 2) Increases the risk of ischemia 3) Delays wound healing 4) Predisposes to hematoma formation

2 1 Chemotherapeutic agents delay wound healing because of their cellular toxicity. 2 Blood pressure medications decrease the amount of pressure required to occlude blood flow to an area, creating a risk for ischemia. 3 Chemotherapeutic agents delay wound healing because of their cellular toxicity. 4 Anticoagulants can lead to extravasation of blood into subcutaneous tissue, predisposing to hematoma formation with minimal pressure or injury.

The nurse is teaching a pregnant woman about the increased oxygen demand that develops during pregnancy. Which client statement indicates that teaching has been effective? 1) "I may need to drink more fluids in order to make more oxygen." 2) "I may need to take an iron supplement so that I am not anemic." 3) "I will need a multivitamin supplement for several months." 4) "I will need to eat more fruits and vegetables."

2 1 Fluids do not increase the amount of oxygen made. 2 During pregnancy, oxygen demand increases dramatically. To compensate, the mother's blood volume increases by 30%. The woman requires additional iron to produce this blood as well as to meet fetal requirements. Failure to meet these iron demands can result in maternal anemia, reducing tissue oxygenation of the mother. 3 A multivitamin may not contain iron, which is needed by the pregnant client. 4 The pregnant client may not ingest an adequate amount of iron that is needed.

A client with pulmonary hypertension and right-sided heart failure has conversational dyspnea and shortness of breath. What is the first action the nurse should take? 1) Review and implement the primary care provider's prescriptions for treatments. 2) Perform a quick physical examination of breathing, circulation, and oxygenation. 3) Gather a thorough medical history, including current symptoms, from the family. 4) Administer oxygen to the patient through a nasal cannula.

2 1 Following a quick assessment, the nurse should then review and implement physician's orders. 2 The first action the nurse should take is to make a quick assessment of the adequacy of breathing, circulation, and oxygenation in order to determine the type of immediate intervention required. The nurse's assessment should include simple questions about current symptoms. 3 A more thorough medical history can be gathered once the patient's oxygenation needs are addressed. 4 Administering oxygen is not appropriate without knowing what treatments the primary care provider has prescribed

Chronic stress may lead directly to cardiovascular disease because of the repeated release of which chemical? 1) Histamine 2) Catecholamines 3) Cortisol 4) Protease

2 1 Histamine is released during an allergic response. 2 The stress response stimulates release of catecholamines from the sympathetic nervous system. This results in increased heart rate and contractility, vasoconstriction, and increased tendency of blood to clot. 3 Cortisol is also released in the stress response, but it is more indirectly related to development of cardiovascular disease through altered glucose, fat, and protein metabolism. 4 Protease is released during an allergic response.

A client is coughing and has bilateral rhonchi throughout the lung fields. Which nursing diagnosis is most appropriate for these assessment findings? 1) Impaired Gas Exchange 2) Ineffective Airway Clearance 3) Ineffective Breathing Pattern 4) Impaired Spontaneous Ventilation

2 1 Impaired Gas Exchange is the appropriate diagnosis if the patient is ventilating adequately but diffusion of gases across the alveolar-capillary membrane is impaired. 2 Ineffective Airway Clearance is the diagnosis used for the inability to maintain a clear airway. 3 Ineffective Breathing Pattern is the diagnosis used to describe inadequate ventilation, such as hypoventilation, hyperventilation, tachypnea, or bradypnea. 4 Impaired Spontaneous Ventilation is the diagnosis used to describe a condition in which a patient, as a result of decreased energy reserves, is unable to maintain breathing adequate to support life.

The nurse is caring for a patient who is experiencing dyspnea. Which position would be most effective if incorporated into the patient's care? 1) Supine 2) Head of bed elevated 80° 3) Head of bed elevated 30° 4) Lying on left side

2 1 Most patients with dyspnea cannot tolerate lying down. 2 Position affects ventilation. An upright or elevated position pulls abdominal organs down, thus allowing maximum diaphragm excursion and lung expansion. 3 The head of the bed should be higher to facilitate breathing. 4 Lying on the left side will not help with this patient's dyspnea

A patient is recovering from surgery for a ruptured appendix. Because the surgeon did not surgically close the wound, what wound healing process will occur? 1) Primary intention 2) Secondary intention 3) Tertiary intention 4) Approximation

2 1 Primary intention healing occurs when a wound is surgically closed. 2 Secondary intention healing occurs when a wound is left open, and it heals from the inner layer to the surface by filling in with beefy red granulation tissue. 3 Tertiary intention healing occurs when a wound that was previously left open to heal by secondary intention is closed by joining the margins of granulation tissue. 4 Approximation is another word for the joining of wound edges.

The nurse prepares material to teach a group of family members on cardiopulmonary resuscitation. Which information should the nurse emphasize in this teaching? 1) Give rescue breaths over 3 seconds. 2) Deliver 100 compressions a minute. 3) Support the head before calling for help. 4) Ensure rescue breathing occurs every 10 compressions.

2 1 Rescue breaths should be provided over 1 full second. 2 Compressions should be provided at the rate of 100 a minute. 3 Supporting the head is not a step in cardiopulmonary resuscitation. 4 Rescue breathing should be at the rate of two breaths every 30 compressions

While assessing a new wound, the nurse notes red, watery drainage. What type of drainage will the nurse document this as? 1) Sanguineous 2) Serosanguineous 3) Serous 4) Purosanguineous

2 1 Sanguineous drainage is bloody. 2 Serosanguineous drainage, a combination of bloody and serous drainage, is most commonly seen with new wounds. 3 Serous drainage is straw colored. 4 Purosanguineous drainage is pus that is red tinged.

A patient recovering from abdominal surgery has an incision has been closed by primary intention, and the staples are intact. To provide more support to the incision site and decrease the risk of dehiscence, what would be appropriate to apply? 1) Steri-Strips 2) Abdominal binder 3) T-binder 4) Paper tape

2 1 Steri-Strips would not be needed for an approximated incision that has intact staples, sutures, or surgical glue. 2 An abdominal binder provides added support to an incision site and decreases the risk of wound dehiscence. 3 A T-binder is used in the perineal area. 4 Paper tape would not be needed for an approximated incision that has intact staples, sutures, or surgical glue.

A client recovering from a respiratory infection is concerned about a new onset of diarrhea. What should the nurse assess in this client? 1) Last use of steroids 2) Amount of vitamin C ingested 3) Frequency of decongestant use 4) Use of over-the-counter antitussives

2 1 Steroids are not known to cause diarrhea. 2 Vitamin C in daily doses of 200 mg or more has not been found to prevent colds, but it has been shown to reduce the length and severity of symptoms. In amounts of 2,000 mg, it may cause diarrhea and gas. 3 Decongestants are not identified as causing diarrhea. 4 Antitussives are not identified as causing diarrhea.

Which part of the electrocardiogram (ECG) tracing represents ventricular repolarization? 1) P wave 2) QRS complex 3) T wave 4) U wave

2 1 The P wave represents the firing of the sinoatrial (SA) node and conduction of the impulse through the atria. In the healthy heart, this leads to atrial contraction. 2 The QRS complex represents ventricular depolarization and leads to ventricular contraction. 3 The T wave represents the return of the ventricles to an electrical resting state so they can be stimulated again (ventricular repolarization). The atria also repolarize, but they do so during the time of ventricular depolarization; thus, they are obscured by the QRS complex and cannot be seen on the ECG complex. 4 The U wave is not always seen on the ECG but may be detected with electrolyte imbalance, such as hypokalemia or hypercalcemia. U waves sometimes occur in response to certain medications (e.g., digitalis, epinephrine). An inverted U wave may occur with ischemia to the cardiac muscle.

Which is the best choice for performing wound irrigation? 1) Water jet irrigation 2) 35-mL syringe with a 19-gauge angiocatheter 3) 5-mL syringe with a 23-gauge needle 4) Bulb syringe

2 1 The water jet irrigation unit would deliver the solution above the recommended pressure range of 4 to 15 pounds per square inch (psi). 2 A 35-mL syringe with a 19-gauge angiocatheter is the best choice for irrigation because it will deliver the irrigation solution at approximately 8 psi. 3 The 5-mL syringe with a 23-gauge needle would deliver the solution above the recommended pressure range of 4 to 15 psi. 4 A bulb syringe is not an appropriate choice because there is an increased risk of aspirating drainage from the wound.

A client requires sedation, intubation, and mechanical ventilation for 2 weeks. What would be an appropriate nursing diagnosis for the client? 1) Risk for Infection related to subcutaneous injuries 2) Risk for Impaired Skin Integrity related to immobility 3) Impaired Tissue Integrity related to ventilator dependency 4) Impaired Skin Integrity related to ventilator dependency

2 1 There is no mention of subcutaneous injuries. 2 The patient is at Risk for Impaired Skin Integrity because of being sedated and will not be able to move self to relieve pressure. 3 Impaired Tissue Integrity is incorrect because there is no supporting evidence for this diagnosis. 4 Impaired Skin Integrity is incorrect because there is no supporting evidence for this diagnosis

A client with a tracheostomy being mechanically ventilated has a pulse oximetry reading of 85%, heart rate of 113 beats/min, and respiratory rate of 30 breaths/min. The client is restless, and crackles and rhonchi are auscultated over both lungs. Which action should the nurse take? 1) Call the respiratory therapist to check the ventilator settings. 2) Provide endotracheal suctioning. 3) Provide tracheostomy care. 4) Notify the physician of the patient's signs of fluid overload.

2 1 There is nothing that indicates the ventilator settings need to be adjusted. 2 Increased pulse and respiratory rates, decreased oxygen saturation, gurgling sounds during respiration, auscultation of adventitious breath sounds, and restlessness are signs that indicate the need for suctioning. Airways are suctioned to remove secretions and maintain patency. The patient's symptoms should subside once the airway is cleared. 3 There is no reason to complete tracheostomy care at this time. 4 The vital signs do not indicate fluid overload.

Which laboratory result should alert the nurse to a potential problem? 1) Na+ = 137 mEq/L 2) K+ = 5.2 mEq/L 3) Ca2+ = 9.2 mg/dL 4) Mg2+ = 1.8 mg/dL

2 1 This is a normal sodium level. 2 A potassium level of 5.2 mEq/L indicates hyperkalemia. 3 This is a normal calcium level. 4 This is a normal magnesium level.

The nurse records a patient's hourly urine output from an indwelling catheter as follows: 0700: 36 mL 0800: 45 mL 0900: 85 mL 1000: 62 mL 1100: 50 mL 1200: 48 mL 1300: 94 mL 1400: 78 mL 1500: 60 mL How should the nurse describe the patient's urine output? 1) Low 2) Within normal limits 3) High 4) Inconclusive

2 1 This patient's urine output is not low. 2 Urine accounts for the greatest amount of fluid loss. Normal urine output for an average-sized adult is approximately 1,500 mL in 24 hr. Urine output varies according to intake and activity but should remain at least 30 to 50 mL per hour. The patient's urine output is within the normal range. This patient has an indwelling catheter, which will result in continual flow of urine. 3 This patient's urine output is not high. 4 There are enough data to make a decision about this patient's urine output.

The nurse notes that a client's troponin level is elevated. What should this finding indicate to the nurse about the client? 1) Has a high risk for cardiovascular disease 2) Experienced a recent myocardial infarction 3) Had a myocardial infarction several months ago 4) Reduced amount of oxygen carrying capacity in the blood

2 1 Troponin level does not determine the risk for cardiovascular disease. 2 Troponin level is used to determine whether a myocardial infarction has occurred. 3 Troponin levels stay elevated for up to 7 days and not several months. 4 Hemoglobin level measures the oxygen carrying capacity in the blood.

When using sterile technique to perform tracheostomy care of a new tracheostomy, which action is correct? 1) Apply sterile gloves. 2) Place the patient in semi-Fowler's position, if possible. 3) Clean the stoma under the faceplate with hydrogen peroxide. 4) Cut a slit in sterile 4 × 4 gauze halfway through to make a dressing.

2 1 Two pairs of sterile gloves are needed: one pair for dressing removal and a clean pair for the rest of the procedure. 2 Semi-Fowler's position promotes lung expansion and prevents back strain for the nurse. 3 The stoma should be cleaned under the faceplate with sterile saline. 4 Never cut a 4 × 4 gauze for the dressing because lint and fibers from the cut edge could enter the trachea and cause respiratory distress.

Which patient is most likely experiencing positive nitrogen balance? A patient admitted: 1) With third-degree burns of his legs. 2) In the sixth month of a healthy pregnancy. 3) From a nursing home who has been refusing to eat. 4) With acute pancreatitis.

2 A positive nitrogen balance typically exists during pregnancy when new tissues are being formed. Patients with burns, malnutrition, and serious illness commonly experience negative nitrogen balance because tissues are lost

Based on the stage of physical development at which toilet training becomes physically possible, for which age would a goal of Achieves toilet training by the end of this month be most appropriate? 1) 18 months 2) 2 1/2 years 3) 3 1/2 years 4) 4 years

2 Between ages 2 and 3 years, a child can typically control defecation, thereby making toilet training possible. Nevertheless, some children, especially boys, may not actually be toilet trained until somewhat later.

When changing a diaper, the nurse observes that a 2-day-old infant has had a green black, tarry stools. What should the nurse do? 1) Notify the physician. 2) Do nothing; this is normal. 3) Give the baby sterile water until the mothers milk comes in. 4) Apply a skin barrier cream to the buttocks to prevent irritation.

2 During the first few days of life, a term newborn passes green black, tarry stools known as meconium. Stools transition to a yellow green color over the next few days. After that, the appearance of stools depends upon the feedings the newborn receives. Sterile water does nothing to alter this progression. Meconium stools are more irritating to the buttocks than other stools because they are so sticky and the skin usually must be rubbed to cleanse it.

The nurse in a long-term care facility is teaching a group of residents about increasing fiber in their diet. Which foods should she explain are high in fiber? 1) White bread, pasta, and white rice 2) Oranges, raisins, and strawberries 3) Whole milk, eggs, and bacon 4) Peaches, orange juice, and bananas

2 Oranges, raisins, and strawberries are high in fiber. White bread, pasta, and white rice are carbohydrates. Whole milk, eggs, and bacon are high in cholesterol. Peaches, orange juice, and bananas are sources of potassium.

A middle-aged patient with a history of alcohol abuse is admitted with acute pancreatitis. This patient will most likely be deficient in which nutrients? 1) Iron 2) B vitamins 3) Calcium 4) Phosphorus

2 Patients who regularly abuse alcohol may be deficient in many nutrients; however, they are commonly deficient in the B vitamins and folic acid. Vitamin A deficiency can be associated with night blindness in heavy drinkers; vitamin D deficiency leads to softening of the bones. Because some alcoholics are deficient in vitamins A, C, D, E, and K and the B vitamins, they experience delayed wound healing. In particular, because vitamin K, the vitamin needed for blood clotting, is commonly deficient in those who regularly abuse alcohol, those patients can have delayed clotting, resulting in excess bleeding. Deficiencies of other vitamins involved in brain function can cause severe neurological damage.

Which laboratory test result most accurately reflects a patients nutritional status? 1) Albumin 2) Prealbumin 3) Transferrin 4) Hemoglobin

2 Prealbumin levels fluctuate daily and give the best indication of the patients immediate nutritional status. Albumin level is not as accurate because the half-life of albumin is 18 to 21 days, causing a delay in detection of nutritional problems. Transferrin, a protein that binds to iron, has a half-life of 8 to 9 days; therefore, it allows for faster detection of protein deficiency than does albumin. However, transferrin is not as fast as prealbumin. Hemoglobin level reflects iron intake or blood loss.

Which nutrient deficiency increases the risk for pressure ulcers? 1) Carbohydrate 2) Protein 3) Fat 4) Vitamin K

2 Protein is necessary for growth and maintenance of body tissues. Protein deficiency places the patient at risk for skin breakdown and pressure ulcer formation. Carbohydrates are the primary fuel of the body. Fat is a source of energy and contains essential nutrients. Vitamin K aids blood clotting.

While performing a physical assessment, the student nurse tells her instructor that she cannot palpate her patients bladder. Which statement by the instructor is best? You should: 1) Try to palpate it again; it takes practice but you will locate it. 2) Palpate the patients bladder only when it is distended by urine. 3) Document this abnormal finding on the patients chart. 4) Immediately notify the nurse assigned to your patient.

2 The bladder is not palpable unless it is distended by urine. It is not difficult to palpate the bladder when distended. The nurse should document her finding, but it is not an abnormal finding. It is not necessary to notify the nurse assigned to the patient.

Patients may be deficient in which vitamin during the winter months? 1) A 2) D 3) E 4) K

2 The body can synthesize vitamin D from a cholesterol compound in the skin when exposed to adequate sunlight. People at risk for vitamin D deficiency are those who spend little time outdoors; older people; and people who live in an institution (e.g., a nursing home). The deficiency can also occur in the winter at northern and southern latitudes, in people who keep their bodies covered (e.g., traditional Muslim women), and in those who use sunscreen. Also, because breast milk contains only small amounts of vitamin D, breastfed infants who are not exposed to enough sunlight are at risk of the deficiency and rickets. There is no seasonal tie to deficiencies in the other fat-soluble vitamins, A, E, and K.

The nurse must irrigate the colostomy of a patient who is unable to move independently. How should the nurse position the patient for this procedure? 1) Semi-Fowlers position 2) Left sidelying position 3) Supine with the head of the bed lowered flat 4) Supine with the head of bed raised to 30 degrees

2 The nurse should position an immobile patient in a left sidelying position to irrigate his colostomy. Semi-Fowlers, supine with the bed lowered flat, and the supine position with the head of bed elevated to 30 degrees are not appropriate positions for colostomy irrigation.

The healthcare team suspects that a patient has an intestinal infection. Which action should the nurse take to help confirm the diagnosis? 1) Prepare the patient for an abdominal flat plate. 2) Collect a stool specimen that contains 20 to 30 ml of liquid stool. 3) Administer a laxative to prepare the patient for a colonoscopy. 4) Test the patient's stool using a fecal occult test.

2 To confirm the diagnosis of an infection, the nurse should collect a liquid stool specimen that contains 20 to 30 ml of liquid stool. An abdominal flat plate and a fecal occult blood test cannot confirm the diagnosis. Colonoscopy is not necessary to obtain a specimen to confirm the diagnosis.

Which daily urine output is within normal limits for a newborn weighing 8 pounds? 1) 288 ml 2) 180 ml 3) 36 ml 4) 18 ml

2 A newborn weighing 8 pounds (3.6 kg) should produce 15 to 60 mL of urine per kilogram per day. If the newborn produces 50 ml/kg/day and weighs 3.6 kg, he will produce a total of 180 ml in 24 hours. The other options are not within normal limits and require further assessment.

A patient is prescribed furosemide (Lasix), a loop diuretic, for treatment of congestive heart failure. The patient is at risk for which electrolyte imbalance associated with use of this drug? 1) Hypocalcemia 2) Hypokalemia 3) Hypomagnesemia 4) Hypophosphatemia

2 Furosemide is a loop diuretic, which causes potassium to pass into the urine. This drug increases the risk for hypokalemia; it does not cause hypocalcemia (low calcium in the blood), hypomagnesemia (low blood magnesium), or hypophosphatemia (low blood phosphorous).

Which outcome is appropriate for the patient who underwent urinary diversion surgery and creation of an ileal conduit for invasive bladder cancer? 1) Patient will resume his normal urination pattern by (target date). 2) Patient will perform urostomy self-care by (target date). 3) Patient will perform self-catheterization by (target date). 4) Patients urine will remain clear with sufficient volume.

2 The most appropriate outcome for this patient is the patient will perform urostomy self-care by a specific date. The patient with an ileal conduit is unable to resume a normal urination pattern; urine, along with mucus, drains continuously from the stoma site, so the urine will not be clear. Plus, the phrase sufficient volume is too vague for an outcome statement. The patient with a continent urostomy inserts a catheter into the stoma to drain urine.

A patient who sustained a spinal cord injury will perform intermittent self-catheterization after discharge. After discharge teaching, which statement by the patient would indicate correct understanding of the procedure? 1) I will need to replace the catheter weekly. 2) I can use clean, rather than sterile, technique at home. 3) I will remember to inflate the catheter balloon after insertion. 4) I will dispose of the catheter after use and get a new one each time.

2 The nurse should inform the patient that clean technique can be used after discharge. The patient should wash his hands before the procedure, then wash the reusable catheter in soap and water, and rinse and store it in a clean dry place. It is not necessary for the patient to use a new catheter for each catheterization. The patient should use a straight catheter; therefore, a balloon is not inflated after insertion. Straight catheters are removed immediately after use.

The nurse instructs a woman about providing a clean catch urine specimen. Which of the following statements indicates that the patient correctly understands the procedure? 1) I will be sure to urinate into the hat you placed on the toilet seat. 2) I will cleanse my genital area from front to back before I collect the specimen midstream. 3) I will need to lie still while you put in a urinary catheter to obtain the specimen. 4) I will collect my urine each time I urinate for the next 24 hours.

2 To obtain a clean catch urine specimen, the nurse should instruct the patient to cleanse the genital area from front to back and collect the specimen midstream. This follows the principle of going from clean to dirty. The nurse should have the ambulatory patient void into a hat (container for collecting the urine of an ambulatory patient) when monitoring urinary output, but not when obtaining a clean catch urine specimen. A urinary catheter is required for a sterile urine specimen, not a clean catch specimen. A 24-hour urine collection may be necessary to evaluate some disorders but a clean catch specimen is a one-time collection.

Which medication will the physician will most likely prescribe to increase urine output in the patient admitted with congestive heart failure? 1) Digoxin 2) Furosemide 3) Lovastatin 4) Atorvastatin

2 loop/ceiling (as per Daco) diuretic, other choices have nothing to do with urinary output.

The nurse instructs the mother of a toddler on safety. What information about the toddler's developmental stage and factors that influence oxygenation should the nurse include? Select all that apply. 1) Frequent, serious respiratory infections 2) Airway obstruction from aspiration of small objects 3) Drowning in small amounts of water around the home 4) Development of asthma 5) Develop shortness of breath with extreme activity

2,3 1. As a toddler's respiratory and immune systems mature, the risk for frequent and serious infections is less than in infanthood. Most children recover from upper respiratory infections without difficulty. 2. Toddlers' airways are relatively short and small and may be easily obstructed, and they often put objects in their mouth as part of exploring their environment, thus increasing their risk for aspiration and airway obstruction. 3. Toddlers are at high risk for drowning in very small amounts of water around the home (e.g., in a bucket of water or toilet bowl). 4. The risk for developing asthma is not significantly influenced by the child's developmental stage. 5. Toddlers do not develop shortness of breath with extreme activity as a developmental issue.

A patient's blood group is B. From which donor groups can this patient receive blood? Select all that apply. 1) A 2) B 3) O 4) AB

2,3 1. Blood group A persons may receive blood from A and O donors. 2. Persons with blood group B can receive blood only from the blood groups B and O. 3. Persons with blood group B can receive blood only from the blood groups B and O. Persons with blood group O may receive blood only from O donors. 4. Those with blood group AB may receive AB, A, B, and O blood.

Which medication would the nurse expect to be prescribed for a patient with heart failure? Select all that apply. 1) Nitrates 2) Beta-adrenergic agents 3) Diuretics 4) Anticoagulants 5) Anticholesterol agents

2,3 1. Nitrates dilate cardiac vessels. 2. Beta-adrenergic agents block stimulation of beta receptors in the heart, lungs, and blood vessels and decrease heart rate, slow conduction through the AV node, and decrease myocardial oxygen demand by reducing myocardial contractility. 3. Diuretics increase removal of sodium and water from the body through increased urine output. Diuretics reduce the volume of circulating blood and prevent accumulation of fluid in the pulmonary circulation. 4. Anticoagulants prevent blood from clotting. 5. Anticholesterol medications (statins) are a class of drugs that protect against coronary artery disease by lowering the level of triglycerides and reduce the production of cholesterol by the liver.

Where in the body is glucose stored? Choose all that apply. 1) Brain 2) Liver 3) Skeletal muscles 4) Smooth muscles

2,3 Human beings store glucose in liver and skeletal muscle tissue as glycogen. Glycogen is converted back into glucose to meet energy needs.

The nurse must administer an enema to an adult patient with constipation. Which of the following would be a safe and effective distance for the nurse to insert the tubing into the patients rectum? Choose all that apply. 1) 2 inches 2) 3 inches 3) 4 inches 4) 5 inches

2,3 When administering an enema, the nurse should insert the tubing about 3 to 4 inches into the patients rectum. Two inches would not be effective because it would not place the fluid high enough in the rectum. Five inches is too much.

Which of the following is/are an appropriate goal(s) for a patient with urinary incontinence? SATA 1) Increase the intake of citrus fruits. 2) Increase daily oral fluids to 8 to 10 glasses per day. 3) Limit daily caffeine intake to less than 100 mg. 4) Engage in high-impact, aerobic exercise.

2,3 The nurse should encourage lifestyle changes such as limiting caffeine intake to fewer than 100 mg per day; limiting intake of alcohol, artificial sweeteners, spicy foods, and citrus fruit; and increasing daily oral fluid intake to 8 to 10 glasses per day. High-impact exercise can be associated with stress incontinence for those with weakened pelvic muscles that support the bladder and urethra.

Which is an appropriate intervention for a patient with hypovolemia? Select all that apply. 1) Teach deep-breathing techniques. 2) Monitor I&O daily. 3) Encourage fluid intake. 4) Monitor electrolyte balance. 5) Measure daily weights.

2,3,4,5 1. Deep-breathing techniques do not address fluid balance; there is no evidence that the patient has a respiratory disorder. 2. Monitoring I&O provides information to evaluate the status of the problem. 3. Encouraging fluid intake helps to correct the problem. 4. It is good to monitor electrolytes because electrolyte imbalance can occur with hypovolemia (although it may not occur at first). 5. Monitoring daily weights is an important intervention for a patient with hypovolemia.

Which nursing intervention reduces the risk of clot formation in the legs? Select all that apply. 1) Keep the patient's hips and knees flexed while the patient is in bed. 2) Apply compression devices (e.g., sequential compression devices [SCDs]). 3) Turn the patient frequently or encourage frequent position changes. 4) Promote adequate hydration by encouraging oral intake. 5) Elevate the patient's legs above the level of the heart.

2,3,4,5 1. Flexion does not prevent clot formation in the legs. 2. Antiembolism stockings and SCDs are frequently used in perioperative patients to promote venous return and prevent clot formation. 3. Turn patients frequently; teach patients to change positions frequently. This prevents vessel injury from prolonged pressure in one position. 4. Promoting adequate hydration helps to keep the blood from becoming viscous ("thick"). Viscous blood clots more readily. 5. Gravity promotes venous return from the feet and legs.

A patient with chronic renal failure will be dialyzed for the first time the following morning. Which is an appropriate nursing intervention for the patient? Select all that apply. 1) Encourage oral fluid intake as desired. 2) Place the patient on strict I&O. 3) Weigh the patient before and after dialysis. 4) Maintain a total fluid restriction of 1,000 mL as prescribed. 5) Monitor laboratory values.

2,3,4,5 1. Fluids are restricted in patients with chronic renal failure because of decreased renal function. Therefore, encouraging oral fluids would not be appropriate. 2. Strict intake and output is an appropriate intervention for this patient. 3. Weighing the patient before and after dialysis is an appropriate intervention. 4. Maintaining a fluid restriction is an appropriate intervention. 5. Monitoring laboratory values is an appropriate intervention.

For which condition is obesity associated with a higher risk of conditions that affect the pulmonary and cardiovascular systems? Select all that apply. 1) Reduced alveolar-capillary gas exchange 2) Lower respiratory tract infections 3) Sleep apnea 4) Hypertension 5) Dyspnea on exertion

2,3,4,5 1. Obesity does not cause reduced alveolar-capillary gas exchange. 2. The risk for respiratory infection increases because lower lung segments are poorly ventilated, and secretions are not removed effectively. 3. Excess neck girth and fat stores in the upper airway often lead to obstructive sleep apnea. 4. Obesity also increases the risk of developing atherosclerosis and hypertension. 5. Large abdominal fat stores press upward on the diaphragm, preventing full chest expansion and leading to hypoventilation and dyspnea on exertion.

Which process occurs during the inflammatory phase of wound healing? Select all that apply. 1) Granulation 2) Hemostasis 3) Epithelialization 4) Inflammation 5) Collagen formation

2,4 1. Granulation occurs in the proliferative stage of wound healing. 2. During the inflammatory phase of wound healing, hemostasis and inflammation occur. After an injury, blood vessels constrict to limit blood loss, and platelets migrate to the site and aggregate to stop bleeding. Together, this results in hemostasis. 3. Epithelialization occurs in the maturation stage of wound healing. 4. Inflammation follows as a defense against infection at the wound site. 5. Collagen formation occurs in the proliferative stage of wound healing.

The nurse examines the electrocardiogram (ECG) tracing of a client and notes tall T waves. What electrolyte imbalance should the nurse suspect? 1) Hypokalemia 2) Hypophosphatemia 3) Hyperkalemia 4) Hypercalcemia

3 1 A flat T wave is associated with hypokalemia. 2 Phosphorous levels do not cause electrocardiogram changes. 3 Potassium levels affect the heart. A tall, peaked T wave on an ECG is associated with hyperkalemia. 4 Calcium levels do not cause electrocardiogram changes

A client in labor after 32 weeks' gestation is eager to deliver. Which client statement indicates that teaching provided about fetal development was effective? 1) "The baby's lungs are well developed now, but he will be at increased risk for SIDS if I deliver early." 2) "We should try to stop this labor now because the baby will be born with sleep apnea if I deliver this early." 3) "If I deliver this early my baby is at risk for respiratory distress syndrome, a condition that can be life threatening." 4) "Thanks for reassuring me; I was pretty sure there isn't much risk to the baby this far along in my pregnancy."

3 1 A premature infant's lungs are not well developed. 2 A premature infant is not at risk for developing sleep apnea. 3 Premature infants (younger than 33 weeks' gestation) are born before the alveolar surfactant system is fully developed. Therefore, they are at high risk for respiratory distress syndrome (RDS). RDS is characterized by widespread atelectasis (collapse of alveoli), usually related to a deficiency of surfactant that keeps air sacs open. 4 There is a risk that the infant's lungs will not be mature.

A patient has an area of nonblanchable erythema on his coccyx. What would be the most important treatment for this patient's stage 1 pressure injury? 1) Transparent film dressing 2) Sheet hydrogel 3) Frequent turn schedule 4) Enzymatic débridement

3 1 A transparent film dressing would protect the area, but a turning schedule is the best intervention. 2 Sheet hydrogel is not indicated for this pressure injury. 3 The patient should be placed on a turn schedule to relieve the pressure. If pressure is not relieved, the wound will worsen. A stage 1 wound is not open, so a dressing is not warranted. 4 Enzymatic débridement is used to remove slough or eschar in an open wound.

When teaching a patient about the healing process of an open wound after surgery, which point should the nurse make? 1) The patient will need to take antibiotics until the wound is completely healed. 2) Because the patient's wound was left open, the wound will likely become infected. 3) The patient will have more scar tissue formation than for a wound closed at surgery. 4) The patient should expect to remain hospitalized until complete wound healing occurs.

3 1 Although open wounds are more prone to infection, this is not an expected outcome, and antibiotics would not necessarily be needed. 2 Although open wounds are more prone to infection, this is not an expected outcome, and antibiotics would not necessarily be needed. 3 Because the wound edges are not approximated, more scar tissue will form. 4 A patient with an open wound should not expect an extended hospital stay if wound care can be provided in the home or an outpatient setting.

The nurse administers an antitussive/expectorant cough preparation to a patient with bronchitis. Which response indicates to the nurse that the medication is effective? 1) The amount of sputum decreases with each dose administered. 2) Cough is completely suppressed, and the patient is able to sleep through the night. 3) Dry, unproductive cough is reduced, but voluntary coughing is more productive. 4) Involuntary coughing produces large amounts of thick yellow sputum.

3 1 An antitussive does not reduce the amount of sputum produced. 2 An antitussive is not a cough suppressant. 3 Antitussives are cough suppressants that reduce the frequency of an involuntary, dry, nonproductive cough. Antitussives are useful for adults when coughing is unproductive and frequent, leading to throat irritation or interrupted sleep. 4 Expectorants help make coughing more productive.

The nurse is preparing to assess a client's cardiovascular system. Which technique should be used when assessing the carotid arteries? 1) Perform capillary refill 2) Palpate for lifts and heaves 3) Auscultate each side for hums 4) Assess skin color and temperature

3 1 Capillary refill determines peripheral blood flow. 2 The precordium is assessed for lifts and heaves. 3 The carotid arteries are auscultated for bruits and hums. 4 Skin color and temperature are assessed for peripheral circulation

The nurse prepares material about blood pressure control for a group of community members. What should the nurse include about baroreceptors? 1) Respond to levels of oxygen in the blood 2) Activate the sympathetic nervous system 3) Maintain blood pressure with body position changes 4) Increase respiration based upon carbon dioxide levels

3 1 Chemoreceptors respond to levels of oxygen in the blood. 2 Chemoreceptors activate the sympathetic nervous system. 3 Baroreceptors located in the walls of the heart and blood vessels are sensitive to pressure changes. The aortic arch and carotid artery baroreceptors are particularly important in the regulation of heart rate and vascular tone. When baroreceptors sense even a small drop in pressure, they send messages to the brainstem centers to stimulate the sympathetic nervous system to increase heart rate and induce vasoconstriction. This mechanism allows people to change positions and maintain blood pressure. 4 Chemoreceptors cause the respiratory rate to change based on carbon dioxide levels in the blood.

Chest percussion and postural drainage would be an appropriate intervention for which condition? 1) Congestive heart failure 2) Pulmonary edema 3) Pneumonia 4) Pulmonary embolus

3 1 Chest physiotherapy is not effective for conditions that do not involve the development of airway secretions, including congestive heart failure. 2 Chest physiotherapy is not effective for conditions that do not involve the development of airway secretions, including pulmonary edema. 3 Chest physiotherapy moves secretions to the large central airways for expectoration or suctioning. 4 Chest physiotherapy is not effective for conditions that do not involve the development of airway secretions, including pulmonary embolus.

A patient has a stage 2 pressure injury that is covered with dry, yellow slough that tightly adheres to the wound. What is the best treatment the nurse could recommend for treating this wound? 1) Dry gauze dressing changed twice daily 2) Nonadherent dressing with daily wound care 3) Hydrocolloid dressing changed as needed 4) Wet-to-dry dressings changed three times a day

3 1 Dry gauze dressing would cover the wound but would not aid in removing the slough. 2 A nonadherent dressing (e.g., Telfa) would cover the wound but would not aid in removing the slough. 3 A hydrocolloid dressing would conform to this area and form a protective layer against friction and bacterial invasion. It would also promote autolytic débridement of the slough and absorb the exudate from the autolysis. 4 A wet-to-dry dressing is a form of mechanical débridement. It would aid in removing the slough but is nonselective; therefore, it could cause damage to healthy tissue as well.

What intervention would be most appropriate for a wound with a beefy red wound bed? 1) Mechanical débridement 2) Autolytic débridement 3) Dressing to keep the wound moist and clean 4) Removal of devitalized tissue and a sterile dressing

3 1 Débridement is not necessary in this situation because there is no devitalized tissue present. 2 Débridement is not necessary in this situation because there is no devitalized tissue present. 3 A red wound indicates active healing, and the best treatment is gentle cleansing and a dressing that will ensure a clean, moist wound environment. 4 Débridement is not necessary in this situation because there is no devitalized tissue present.

A client is prescribed continuous cardiac monitoring. Which action should the nurse take to ensure an accurate electrocardiogram tracing? 1) Select electrode placement sites over bony prominences. 2) Apply the electrodes immediately after cleansing the skin, before the alcohol evaporates. 3) Before applying the electrodes, rub the placement sites with gauze until the skin reddens. 4) Ensure that the gel on the back of the electrodes is dry.

3 1 Electrodes should be placed over soft tissues or close to bone in order to obtain accurate waveforms. Sites over bony prominences, thick muscles, and skinfolds can produce artifact; therefore, they should not be used. 2 Alcohol removes skin oils that may prevent the electrodes from adhering. However, the alcohol should be allowed to dry before the electrodes are placed. 3 Rubbing the skin with gauze or a washcloth removes dead skin cells and promotes better electrical contact. 4 A dry electrode will not conduct electrical activity; gel should not be dry.

The nurse notes that a client is prescribed low molecular weight heparin injections. What should the nurse realize is the reason for this medication? 1) Maintain a normal heart rhythm 2) Reduce the development of hypertension 3) Prevent development of an asymptomatic blood clot 4) Maximize perfusion of blood to the lower extremities

3 1 Heparin is not used to maintain cardiac rhythm. 2 Heparin is not an antihypertensive medication. 3 Hospitalized patients are at particularly high risk for clot formation and may develop an asymptomatic DVT and die of pulmonary embolism even before the diagnosis is suspected. For this reason, The Joint Commission (2013) identified anticoagulant therapy as the "number one patient safety practice" for hospitalized patients. 4 Heparin does not ensure perfusion to the lower extremities.

Which information provides the most reliable data about the effectiveness of airway suctioning? 1) The amount, color, consistency, and odor of secretions 2) The patient's tolerance for the procedure 3) Breath sounds, vital signs, and pulse oximetry before and after suctioning 4) The number of suctioning passes required to clear secretions

3 1 Information about the amount and appearance of secretions provides data about the likelihood of airway infection and/or inflammation. 2 Data about the patient's tolerance of suctioning provide information about the patient's overall condition. 3 Breath sounds, vital signs, and oxygen saturation levels before and after suctioning provide data about the effectiveness of suctioning. 4 The number of suctioning passes required to clear the secretions provides information about the amount of secretions present.

The nurse is caring for a patient with a medical diagnosis of hypernatremia. The following prescriptions are written in the client's electronic health record. Which one should the nurse question? 1) Administer an IV of D5W at 125 mL/hr. 2) Strict I&O monitoring. 3) Restrict oral intake to 900 mL every 24 hr. 4) Monitor serum electrolytes every 4 hr.

3 1 Infusing D5W IV fluid is appropriate, as this solution does not contain sodium. Hydrating the patient with D5W would reduce the serum sodium level. 2 Strict I&O monitoring would ensure that the patient is safely rehydrated. 3 Restricting the oral intake of a patient with hypernatremia (Na+ greater than 145 mEq/L) would lead to further elevation in the serum sodium level. 4 Laboratory evaluation of electrolytes every 4 hr would ensure that the patient is safely rehydrated.

Which is an applicable goal/outcome for the client with a stage 4 pressure injury? 1) Client will maintain intact skin throughout hospitalization. 2) Client will limit pressure to wound site throughout treatment course. 3) Wound will close with no evidence of infection within 6 weeks. 4) Wound will improve prior to discharge as evidenced by a decrease in drainage.

3 1 Intact skin throughout hospitalization is not realistic with a stage 4 pressure injury. 2 Limiting pressure to a wound site is incorrect because total pressure relief must be provided to the area. 3 The goal for any wound is for healing to take place with no complications (such as infection). 4 Improved wound drainage before discharge is not a realistic expectation for a stage 4 pressure injury.

A patient has just had a chest tube inserted to dry-seal suction drainage. Which is a correct nursing intervention for maintenance? 1) Keep the head of the bed flat for 6 hours. 2) Immobilize the patient's arm on the affected side. 3) Keep the drainage system lower than the insertion site. 4) Drain condensation into the humidifier when it collects in the tubing.

3 1 Maintain patient in semirecumbent position (head of bed elevated 30 to 45 degrees), not flat. This is extremely important to promote lung expansion, reduce gastric reflux, and prevent ventilator-associated pneumonia if the person is being mechanically ventilated. 2 Encourage the patient to move the arm on the affected side; if he cannot, perform passive range of motion. 3 The drainage system must be below the insertion site to prevent fluid flowing back into the pleural cavity and compromising the patient's respiratory status. 4 The fluid should not be drained into the humidifier because the patient's secretions may have contaminated it.

A client receives treatment for heart failure. Which information in the client's health history contributed to the development of this health problem? 1) Plays golf twice a week 2) Works as a home contractor 3) Ingests a six pack of beer a day 4) Coaches Little League in the summer

3 1 Physical activity helps reduce the risk of developing heart disease. 2 A physically active job helps prevent the development of heart disease. 3 Chronic alcohol abuse causes fatty infiltration of the heart muscle, thrombi in the coronary arteries, heart enlargement, and dysrhythmias, all of which can ultimately lead to heart failure. 4 Physical activity helps reduce the risk of developing heart disease.

Which electrolyte is the primary regulator of fluid volume? 1) Potassium 2) Calcium 3) Sodium 4) Magnesium

3 1 Potassium is a key electrolyte in cellular metabolism. 2 Calcium is responsible for bone health and neuromuscular and cardiac functions. It is also an essential factor in blood clotting. 3 Sodium is the major cation in the extracellular fluid (ECF). Its primary function is to regulate fluid volume. When sodium is reabsorbed in the kidney, water and potassium are also reabsorbed, thereby maintaining ECF volume. 4 Magnesium is a mineral used in more than 300 biochemical reactions in the body.

Which client does the nurse recognize as being at greatest risk for pressure injuries? 1) Infant with skin excoriations in the diaper region 2) Young adult with diabetes in skeletal traction 3) Middle-aged adult with quadriplegia 4) Older adult requiring use of assistive device for ambulation

3 1 The infant with disruption to the skin from diaper rash is at risk for skin infection but not for a pressure injury. 2 The young adult with diabetes is at increased risk for delayed wound healing but not likely for a pressure injury because he would shift weight in bed and respond to discomfort of pressure on a bony site. 3 The client at greatest risk for pressure injury is the one with a lack of sensory perception at the site (e.g., quadriplegia). 4 The older adult is normally at risk for pressure injury, but when mobile, even with an assistive device, the risk is minimal

A 62-year-old man with emphysema does not understand the need to stop smoking at this age because lung problems already exist. Which would be the best response to his statement? 1) "You should quit so your family does not get sick from exposure to secondhand smoke." 2) "You will need to use oxygen, but remember it is a fire hazard to smoke with oxygen in your home." 3) "Once you stop smoking, your body will begin to repair some of the damage to your lungs." 4) "You should ask your primary care provider for a prescription for a nicotine patch to help you quit."

3 1 The suggestion that the patient's family will become ill appears to be a scare tactic, which can be seen as coercive and would not be effective in motivating the patient to stop smoking. 2 The suggestion that oxygen is a fire hazard appears to be a scare tactic, which can be seen as coercive and would not be effective in motivating the patient to stop smoking. 3 The nurse's response should focus on correcting the patient's misinformation rather than on convincing him to stop smoking. Once a person stops smoking, the body begins to repair the damage. During the first few days, the person will cough more as the cilia begin to clear the airways. The coughing then subsides, and breathing becomes easier. Even long-time smokers can benefit from smoking cessation. 4 Although asking the primary care provider for a prescription may help the patient to stop smoking, it does not address the fact that he does not understand why he needs to stop smoking even though lung problems already exist.

A patient has been vomiting for 2 days, has not been able to eat or drink anything during this time, and has not urinated for 12 hours. Physical examination reveals the following: T = 99.6°F (37.6°C) orally; P = 110 beats/min weak and thready; BP = 80/52 mm Hg. Skin and mucous membranes are dry, there is decreased skin turgor, and the patient is experiencing weakness. The following are the most recent laboratory results: Sodium: 138 mEq/L Potassium: 3.7 mEq/L Calcium: 9.2 mg/dL Magnesium: 1.8 mg/dL Chloride: 99 mEq/L BUN: 29 mg/dL Which is an appropriate nursing diagnosis for this patient? 1) Impaired Gas Exchange related to ineffective breathing 2) Excess Fluid Volume related to limited fluid output 3) Deficient Fluid Volume related to abnormal fluid loss 4) Electrolyte Imbalance related to decreased oral intake

3 1 There are no data to support the diagnosis of Impaired Gas Exchange. 2 The symptoms are not consistent with the diagnosis of Excess Fluid Volume. 3 Vomiting has made this patient hypovolemic. Deficient Fluid Volume is the appropriate diagnosis. 4 Electrolyte Imbalance is not a nursing diagnosis.

The nurse suspects that a patient's intravenous solution has infiltrated into the tissues. What action should the nurse take first? 1) Aspirate, then inject 0.5 mL normal saline. 2) Restart the IV line in a different vein. 3) Stop the infusion immediately. 4) Notify the primary care provider.

3 1 There is no point in injecting saline because doing so puts even more fluid in the tissues. Injecting fluid to try to clear a clot from the catheter is not recommended because of the possibility of causing an embolism. 2 Once the infusion is stopped, the nurse must assess whether the patient needs additional IV therapy. 3 The nurse should first stop the infusion to avoid further tissue trauma. 4 The nurse may need to inform the primary care provider if she is unable to find a new IV site or if she believes the patient no longer needs an IV.

A patient has a continuous IV infusion at 60 mL/hr. The right hand IV has infiltrated and the nurse has started a new IV on the left forearm. Which intervention should the nurse also perform? 1) Elevate the patient's left forearm. 2) Schedule daily dressing changes to the new IV site. 3) Change the administration set. 4) Place the patient in Fowler's position.

3 1 There is no reason to elevate the patient's left forearm. 2 IV dressings are usually changed every 72 to 96 hours when the IV site is rotated. 3 Reusing an IV set from a previous site increases the risk of contamination. 4 There is no reason to place the patient in Fowler's position.

A client is admitted to the emergency department (ED) in respiratory distress. The results of his arterial blood gases are pH = 7.30 PCO2 = 40 HCO3 = 19 mEq/L PO2 = 80. How should the nurse interpret these results? 1) Respiratory acidosis with normal oxygen levels 2) Respiratory alkalosis with hypoxia 3) Metabolic acidosis with normal oxygen levels 4) Metabolic alkalosis with hypoxia

3 1 These results are not seen in respiratory acidosis. 2 These results are not seen in respiratory alkalosis. The oxygen level is within normal range. 3 The pH is acidotic. The HCO3 of 19 mEq/L is low and has moved in the same direction as the pH, indicating a metabolic disorder. The PCO2 is within normal range with no signs of compensation. The PO2 level is normal. 4 These results are not seen in metabolic alkalosis.

A healthcare provider prescribes 250 mL of 0.9% sodium chloride to be infused over 2 hours. A microdrip infusion set is being used. What is the drip rate (drops/min) that the nurse should monitor? 1) 60 2) 75 3) 125 4) 250

3 1 This is an incorrect calculation. 2 This is an incorrect calculation. 3 Calculate the drip rate by multiplying the number of milliliters to be infused per hour (hourly rate) by the drop factor in drops/mL, divided by 60 minutes. An infusion of 250 mL in 2 hours results in an hourly rate of 125 mL/hr. 125 (mL/hr) × 60 (drops/mL) = 125 drops/min 60 min 4 This is an incorrect calculation.

What is the function of the stratum corneum? 1) Provides insulation for temperature regulation 2) Provides strength and elasticity to the skin 3) Protects the body against the entry of pathogens 4) Continually produces new skin cells

3 1. The subcutaneous layer is composed of adipose and connective tissue that provide insulation, protection, and an energy reserve (adipose). 2. The dermis is composed of irregular fibrous connective tissue that provides strength and elasticity to the skin. 3. The stratum corneum is the outermost layer of the epidermis and is composed of numerous thicknesses of dead cells. Functioning as a barrier to the environment, it restricts water loss, prevents entry of fluids into the body, and protects the body against the entry of pathogens and chemicals. 4. The stratum germinativum is the innermost layer of the skin that produces new cells, pushing older cells toward the skin surface.

A patient who underwent surgery 24 hours ago is prescribed a clear liquid diet. The patient asks for something to drink. Which item may the nurse provide for the patient? 1) Tea with cream 2) Orange juice 3) Gelatin 4) Skim milk

3 A clear liquid diet consists of water; tea (without cream); coffee; broth; clear juices, such as apple, grape, or cranberry; popsicles; carbonated beverages; and gelatin. Skim milk, tea with cream, and orange juice are included in a full liquid diet.

Which is a key treatment intervention for the patient admitted with diverticulitis? 1) Antacid 2) Antidiarrheal agent 3) Antibiotic therapy 4) NSAIDs

3 A key treatment for diverticulitis (an infected diverticulum) is antibiotic therapy; if antibiotic therapy is ineffective, surgery may be necessary. Antacids, antidiarrheal agents, and NSAIDs are not indicated for treatment of diverticulitis.

A nurse is teaching wellness to a women's group. The nurse should explain the importance of consuming at least how much fluid to promote healthy bowel function (assume these are 8-ounce glasses)? 1) 2 to 4 glasses a day 2) 4 to 6 glasses a day 3) 6 to 8 glasses a day 4) 8 to 10 glasses a day

3 A minimum of 6 to 8 glasses of fluid should be consumed each day to promote healthy bowel function.

A patient with a skin infection is prescribed cephalexin 500 mg orally q 12 hours. The patient complains that the last time he took this medication, he had frequent episodes of loose stools. Which recommendation should the nurse make to the patient? 1) Stop taking the drug immediately if diarrhea develops. 2) Take an antidiarrheal agent such as diphenoxylate. 3) Consume yogurt daily while taking the antibiotic. 4) Increase your intake of fiber until the diarrhea stops.

3 Antibiotics such as cephalexin, given to combat infection, decrease the normal flora in the colon that cause diarrhea. Bacterial populations can be maintained by encouraging the patient to consume yogurt daily while taking the drugs. Diarrhea is a common adverse effect of antibiotics; stopping the drug is not necessary. The patient should not be encouraged to take an antidiarrheal agent at this time. Increasing the intake of fiber combats constipation, not diarrhea.

During the day shift, a patients temperature measures 97F (36.1C) orally. At 2000, the patients temperature measures 102F (38.9C). What effect does this rise in temperature have on the patients basal metabolic rate? 1) Increases the rate by 7% 2) Decreases the rate by 14% 3) Increases the rate by 35% 4) Decreases the rate by 28%

3 Basal metabolic rate increases 7% for each degree Fahrenheit (0.56C); therefore, this patients temperature rise is an increase of 35%.

A patients 2:1 parenteral nutrition container infuses before the pharmacy prepares the next container. This places the patient at risk for which complication? 1) Sepsis 2) Pneumothorax 3) Hypoglycemia 4) Thrombophlebitis

3 Because of the high glucose content of 2:1 parenteral nutrition, any interruption in therapy places the patient at risk for hypoglycemia. A PN of this type should not be discontinued abruptly, but rather over several (as many as 48) hours to prevent a sudden drop in blood sugar. Hypoglycemia is unlikely to occur with a 3:1 solution (containing lipids), as the final concentration of glucose is less than 10%. Sepsis is a complication that can occur if a break in aseptic technique occurs during therapy. Pneumothorax can occur as a result of central venous catheter insertion. Central venous catheters are typically employed for parenteral nutrition. Thrombophlebitis is a complication of central venous catheter use.

Which polysaccharide is stored in the liver? 1) Insulin 2) Ketones 3) Glycogen 4) Glucose

3 Humans store glucose in the liver as polysaccharides, known as glycogen. Glycogen can then be converted back into glucose to meet energy needs through a process known as glycogenolysis. If fats must be used for energy, they are converted directly into ketones. Insulin is a pancreatic hormone that promotes the movement of glucose into cells

The nurse is instructing a patient about performing home testing for fecal occult blood. The nurse should explain that ingestion of which substance may cause a false-negative fecal occult blood test? 1) Vitamin D 2) Iron 3) Vitamin C 4) Thiamine

3 Ingestion of vitamin C can produce a false-negative fecal occult blood test; ingestion of vitamin D, iron, and thiamine does not. Iron can lead to a false-positive result.

A patient has a colostomy in the descending (sigmoid) colon and wishes to control bowel evacuation and possibly stop wearing an ostomy pouch. To help achieve this goal, nurse should teach the patient to: 1) Call the primary care provider if the stoma becomes pale, dusky, or black. 2) Limit the intake of gas-forming foods such as cabbage, onions, and fish. 3) Irrigate the stoma to produce a bowel movement on a schedule. 4) Avoid returning to the use of an ostomy appliance if he becomes ill.

3 Patients with an ostomy in the descending or sigmoid colon may use colostomy irrigation as a means to control and schedule bowel evacuation and possibly eliminate the need to wear an ostomy pouch. Limiting the intake of gas-forming foods is a good idea from a social perspective; however, it does not help achieve the goal of having regular BMs and eliminating the need to wear a pouch. When illness occurs, it may be difficult to control the output, so the patient can use an ostomy appliance. This will not make it more difficult to schedule the BMs after the illness passes.

Which nutritional goal is appropriate for a patient newly diagnosed with hypertension? The patient will: 1) Limit his intake of protein. 2) Avoid foods containing gluten. 3) Restrict his use of sodium. 4) Limit his intake of potassium-rich foods.

3 Patients with hypertension should limit their intake of sodium. Those with liver disease should control their protein intake. Patients with renal disease must limit their intake of potassium-rich foods. Patients with celiac disease should avoid foods containing gluten.

While addressing a community group, the nurse explains the importance of replacing saturated fats in the diet with mono- and polyunsaturated fats. She emphasizes that doing so greatly reduces the risk of which complication? 1) Kidney failure 2) Liver failure 3) Stroke 4) Lung cancer

3 Replacing saturated fats in the diet with mono- and polyunsaturated fats reduces the risk of heart disease and stroke, not kidney failure, liver failure, or lung cancer.

Which of the following goals is appropriate for a patient with a nursing diagnosis of Constipation? The patient increases the intake of: 1) Milk and cheese. 2) Bread and pasta. 3) Fruits and vegetables. 4) Lean meats.

3 The nurse should encourage the patient to increase his intake of foods rich in fiber because they promote peristalsis and defecation, thereby relieving constipation. Low-fiber foods, such as bread, pasta, and other simple carbohydrates, as well as milk, cheese, and lean meat, slow peristalsis.

A patient with type 1 diabetes is admitted with hyperglycemia and associated acidosis. The presence of which alternative fuel in the body is responsible for the acidosis? 1) Glycogen 2) Insulin 3) Ketones 4) Proteins

3 When fats are converted to ketones for use as alternative fuel, as in diabetic ketoacidosis when glucose cannot by used by the cells, the acidity of the blood rises, leading to the acidosis. Glycogen is converted to glucose to meet energy needs. Insulin, a pancreatic hormone, promotes the movement of glucose into cells for use. Proteins would not be used for fuel as long as fats were available

The nurse notes that a patients indwelling urinary catheter tubing contains sediment and crusting at the meatus. Which action should the nurse take? 1) Notify the provider immediately. 2) Flush the catheter tubing with saline solution. 3) Replace the indwelling urinary catheter. 4) Encourage fluids that increase urine acidity.

3 The catheter needs to be changed when sediment collects in the tubing or catheter and crusting at the meatus occurs. It is not necessary to notify the provider immediately. The nurse should not flush the catheter tubing. The patient should be encouraged to consume fluids that increase urine acidity to prevent urinary tract infection; however, it will not help clear the catheter tubing of sediment.

What is the best technique for obtaining a sterile urine specimen from an indwelling urinary catheter? 1) Use antiseptic wipes to cleanse the meatus prior to obtaining the sample. 2) Briefly disconnect the catheter from the drainage tube to obtain sample. 3) Withdraw urine through the port using a needleless access device. 4) Obtain the urine specimen directly from the collection bag.

3 To obtain a specimen from an indwelling catheter, insert the needleless access device with a 20- or 30-mL syringe into the specimen port, and aspirate to withdraw the amount of urine you need. Wiping the meatus with an antiseptic material helps to minimize contamination for a clean catch voided specimen, not a sample collected from a closed system such as an indwelling catheter system. Never disconnect the catheter from the drainage tube to obtain a sample. Interrupting the system creates a portal of entry for pathogens, thereby increasing the risk of contamination. Do not take the specimen from the collection bag because that urine may be several hours old.

The nurse is caring for a patient who underwent a bowel resection 2 hours ago. His urine output for the past 2 hours totals 50 mL. Which action should the nurse take? 1) Do nothing; this is normal postoperative urine output. 2) Increase the infusion rate of the patients IV fluids. 3) Notify the provider about the patients oliguria. 4) Administer the patients routine diuretic dose early.

3 50 mL in two hours is not normal output. The kidneys typically produce 60 ml of urine per hour. Therefore, the nurse should notify the provider when the patient shows diminished urine output (oliguria). Patients who undergo abdominal surgery commonly require increased infusions of IV fluid during the immediate postoperative period. The nurse cannot provide increased IV fluids without a providers order. The nurse should not administer any medications before the scheduled time without a prescription. The provider may hold the patients scheduled dose of diuretic if he determines that the patient is experiencing deficient fluid volume.

The nurse is teaching an older female patient how to manage urge incontinence at home. What is the first-line approach to reducing involuntary leakage of urine? 1) Insertion of a pessary 2) Intermittent self-catheterization 3) Bladder training 4) Anticholinergic medication

3 The goal of bladder training is to enable the patient to hold increasingly greater volumes of urine in the bladder and to increase the interval between voiding. This involves patient teaching; scheduled voiding; and self-monitoring using a voiding diary. In addition to teaching the mechanisms of urination, teach distraction and relaxation strategies to help inhibit the urge to void. Other techniques include deep breathing and guided imagery. A pessary is an incontinence device that is inserted into the vagina to reduce organ prolapse or pressure on the bladder. Clean intermittent self-catheterization is a good option for managing incontinence that is resistant to conservative measure, such as bladder training, Kegel exercises, lifestyle modification, and medication. Anticholinergic medication can be a highly effective for improving urinary incontinence. However, more conservative measures, such as timed voiding and Kegel exercises, are recommended first.

Which task can the nurse safely delegate to the nursing assistive personnel? 1) Palpating the bladder of a patient who is unable to void 2) Administering a continuous bladder irrigation 3) Providing indwelling urinary catheter care 4) Obtaining the patients history and physical assessment

3 The nurse can safely delegate indwelling urinary catheter care to a nursing assistive personnel who is adequately trained to do so. Palpating the bladder, administering a continuous bladder irrigation, and obtaining the patients history and physical assessment involve the critical thinking skills of a professional nurse.

What position should the patient assume before the nurse inserts an indwelling urinary catheter? 1) Modified Trendelenburg 2) Prone 3) Dorsal recumbent 4) Semi-Fowlers

3 The nurse should have the patient lie supine with knees flexed, feet flat on the bed (dorsal recumbent position). If the patient is unable to assume this position, the nurse should help the patient to a side-lying position. Modified Trendelenburg position is used for central venous catheter insertion. Prone position is sometimes used to improve oxygenation in patients with adult respiratory distress syndrome. Semi-Fowlers position is used to prevent aspiration in those receiving enteral feedings.

Which is a risk assessment tool used in the United States to evaluate a patient's risk for pressure ulcers? Select all that apply. 1) Pressure ulcer healing chart 2) PUSH tool 3) Braden scale 4) Norton scale 5) Glasgow scale

3,4 1. The Pressure Injury Healing Chart is part of the PUSH tool, which is used to monitor the progression of a pressure ulcer. 2. The PUSH tool provides a comprehensive means of reporting the progression of a pressure injury. Surface area, exudate, and type of wound tissue are scored and totaled. 3. The Braden scale is a tool used to predict the risk of developing a pressure injury. Evaluation is based on six areas (indicators): sensory perception, moisture, activity, mobility, nutrition, and friction or shear. 4. The Norton scale is a tool used to assess the risk for pressure injury based on the patient's physical condition, mental state, activity, mobility, and incontinence. 5. The Glasgow scale is used to evaluate level of consciousness.

A patient has been diagnosed with hypovolemia. Which hydration prescription should the nurse question? Select all that apply 1) 0.9% (normal) saline at 100 mL/hr 2) Lactated Ringer's solution at 100 mL/hr 3) Total parenteral nutrition solution at 100 mL/hr 4) D5W solution at 100 mL/hr 5) D5 0.9% NaCl at 100 mL/hr

3,4,5 1. 0.9% (normal) saline is an isotonic fluid that remains inside the intravascular space, thus increasing volume. 2. Lactated Ringer's is an isotonic fluid that remains inside the intravascular space, thus increasing volume. 3. Total parenteral nutrition is a hypertonic fluid used to provide nutrition for the patient who cannot meet caloric needs by eating or enteral nutrition. 4. The D5W is a hypotonic solution that would pull body water from the intravascular compartment into the interstitial fluid compartment. 5. D5 0.9% NaCl is a hypertonic fluid that pulls fluids and electrolytes from the intracellular and interstitial compartments into the intravascular compartment

Which intervention is likely to reduce the risk of postoperative atelectasis? Select all that apply. 1) Administer bronchodilators. 2) Apply low-flow oxygen. 3) Encourage coughing and deep breathing and coughing. 4) Administer pain medication. 5) Assist to move and reposition in bed.

3,4,5 1. Bronchodilators are not routinely prescribed as postoperative medication to prevent atelectasis. 2. Low-flow oxygen will not prevent atelectasis. 3. Once pain is controlled, encouraging patients to deep-breathe and cough will help mobilize secretions and prevent atelectasis. 4. Pain alters the rate and depth of respirations. Often, patients in pain breathe shallowly, which puts them at risk for atelectasis. Regularly assess all patients for pain. 5. Movement and repositioning helps prevent postoperative atelectasis.

A patient with chronic obstructive pulmonary disease (COPD) is prescribed O2 at 24% FIO2. What is the most appropriate oxygen delivery method for this patient? 1) Nonrebreather mask 2) Nasal cannula 3) Partial rebreather mask 4) Venturi mask

4 Rebreather masks are used when high concentrations of oxygen are required. 2 A nasal cannula administers oxygen in liters per minute and does not allow administration of a precise FIO2. 3 Rebreather masks are used when high concentrations of oxygen are required. 4 The Venturi mask is capable of delivering 24% to 50% FIO2. The cone-shaped adapter at the base of the mask allows a precise FIO2 to be delivered. This is very useful for patients with chronic lung disease

A patient has a stage 1 pressure injury on the left heel. What is the initial treatment for this pressure ulcer? 1) Antibiotic therapy for 2 weeks 2) Normal saline irrigation of the ulcer daily 3) Débridement to the left heel 4) Elevation of the left heel off the bed

4 1 A stage 1 pressure injury does not need topical antibiotic therapy. 2 A stage 1 pressure injury does not need to be irrigated. 3 A stage 1 pressure injury does not need to be debrided. 4 Pressure ulcers are caused by pressure to an area that restricts blood flow, causing ischemia to underlying tissue. The primary treatment is to relieve the pressure, thus improving blood flow. Elevating the patient's left heel off the bed would relieve pressure to this area.

A patient has a pressure injury on the coccyx measuring 5 cm by 3 cm that is covered with eschar. How should the nurse classify this wound? 1) Stage 2 pressure injury 2) Stage 3 pressure injury 3) Stage 4 pressure injury 4) Unstageable pressure injury

4 1 A stage 2 pressure injury does not have eschar. 2 A stage 3 pressure injury does not have eschar. 3 A stage 4 pressure injury does not have eschar. 4 An eschar is a black, leathery covering made up of necrotic tissue. An ulcer covered in eschar cannot be classified using a staging method because it is impossible to determine the depth.

When would the nurse know that care provided to a stage 2 pressure injury has been effective? 1) The ulcer is completely healed with minimal scarring. 2) The patient reports no pain at the site. 3) A minimal amount of drainage is noted. 4) The wound bed contains 100% granulated tissue

4 1 A wound can heal, leaving a scar. 2 No pain does not indicate proper healing is occurring. 3 Minimal drainage does not indicate proper healing is occurring. 4 A healing wound contains granulating tissue.

Which is a characteristic of chronic wounds? 1) Are full-thickness wounds 2) Result from pressure 3) Are usually infected 4) Exceed the typical healing time

4 1 Acute or chronic wounds can be full thickness. 2 Acute or chronic wounds can be the result of pressure. 3 Chronic wounds are considered contaminated but not necessarily infected. 4 Wounds that exceed the anticipated length of recovery are classified as chronic wounds.

Which outcome indicates that the plan of care for a client with diabetes and severe peripheral neuropathy was effective? 1) Begins an aggressive exercise program 2) Follows a diet plan of 1,200 calories per day 3) Is fitted for deep-depth diabetic footwear 4) Remains free of foot wounds

4 1 An aggressive exercise program would not be appropriate for a client with severely diminished sensation in the feet. 2 A 1,200-calorie diet would be inadequate for most clients. 3 Being fitted for diabetic footwear is an intervention rather than a goal. 4 The nurse will know his plan of care is effective when the client's feet remain free of wounds.

The nurse working in the emergency department is preparing heat therapy for one of the patients in the unit. Which one is it most likely to be? 1) Actively bleeding 2) Swollen, tender insect bite 3) Sprained ankle 4) Lower back pain

4 1 Applying heat promotes vasodilation and reduces blood thickness (viscosity) and leaky capillaries, all of which would be harmful to the patient who is actively bleeding. It can lead to a drop in blood pressure. 2 Heat should not be applied to a site with inflammation because it can increase edema to the site. 3 Heat should not be applied to a joint injury with swelling because it can increase edema to the site. 4 Heat therapy is used to relieve stiffness and discomfort commonly associated with musculoskeletal soreness. Heat causes dilation of the blood vessels and improves delivery of oxygen and nutrients to the tissues. It promotes relaxation and is used to aid in the healing process

A client asks why aspirin has been prescribed even though pain is not being experienced. What should the nurse respond to this client? 1) "It dilates blood vessels." 2) "It dissolves clots in the body." 3) "It keeps blood pressure within normal range." 4) "It stops the mechanism that makes blood clots."

4 1 Aspirin is not a vasodilator. 2 Aspirin does not dissolve clots. 3 Aspirin is not an antihypertensive. 4 Aspirin blocks the production of prostaglandin, a hormone-like substance that activates the formation of blood clots, signals an injury, and triggers pain. This helps reduce the risk of heart attack or stroke from clot-blocked arteries, decreases pain and inflammation, and reduces the short-term risk of death among people suffering from heart attacks.

A client with a closed head injury has a respiratory pattern that progressively increases and then decreases in depth, followed by a period of apnea. What is this client's breathing pattern? 1) Biot's breathing 2) Kussmaul's respirations 3) Sleep apnea 4) Cheyne-Stokes respirations

4 1 Biot's respirations are irregular respirations of variable depth (usually shallow), alternating with periods of apnea. 2 Kussmaul's respirations are regular but increased in rate and abnormally deep respirations. 3 Sleep apnea is not a type of breathing pattern. 4 This respiratory pattern is known as Cheyne-Stokes respirations. It is often associated with damage to the medullary respiratory center or high intracranial pressure due to brain injury.

Which blood level normally provides the primary stimulus for breathing? 1) pH 2) Oxygen 3) Bicarbonate 4) Carbon dioxide

4 1 Blood pH does not provide the stimulus for breathing. 2 Oxygen level does not provide the stimulus for breathing. 3 Bicarbonate level does not provide the stimulus for breathing. 4 Carbon dioxide (CO2) level provides the primary stimulus to breathe. High CO2 levels stimulate breathing to eliminate the excess CO2. A secondary, although important, drive to breathe is hypoxemia. Low blood O2 levels stimulate breathing to bring more oxygen into the lungs.

A client has a nursing diagnosis of Ineffective Breathing Pattern identified on the care plan. What should the nurse expect when assessing this client? 1) Coughing 2) Cold extremities 3) Adventitious breath sounds 4) Respiratory rate of 8 breaths/min

4 1 Coughing would be the finding for the diagnosis of Ineffective Airway Clearance. 2 A variety of diagnoses would be appropriate for the finding of cold extremities. 3 Adventitious breath sounds would be the finding for the diagnosis of Ineffective Airway Clearance. 4 An Ineffective Breathing Pattern is used to describe inadequate ventilation, such as hypoventilation, hyperventilation, tachypnea, or bradypnea. A breathing rate of 8 breaths/min is what the nurse should expect to assess in this client.

The nurse notes that a client has had significant diarrhea over the past several days. Which level in the client's arterial blood gas result will be most affected? 1) pH 2) PO2 3) PCO2 4) HCO3

4 1 Diarrhea does not directly affect the blood pH. 2 Diarrhea does not directly affect the oxygen level. 3 Diarrhea does not directly affect the carbon dioxide level. 4 HCO3 maintains acid-base balance by functioning as the primary buffer in the body. Levels rise and fall to maintain pH. Bicarbonate is lost through diarrhea.

Which process requires energy to maintain the unique composition of extracellular and intracellular compartments? 1) Diffusion 2) Osmosis 3) Filtration 4) Active transport

4 1 Diffusion is a passive process. 2 Osmosis is a passive process. 3 Filtration is a passive process. 4 Active transport occurs when molecules move across cell membranes from an area of low concentration to an area of high concentration. Active transport requires energy expenditure for the movement to occur against a concentration gradient. In the presence of ATP, the sodium-potassium pump actively moves sodium from the cell into the extracellular fluid. Active transport is vital for maintaining the unique composition of both the extracellular and intracellular compartments.

A patient's vital signs prior to a blood transfusion were T = 97.6°F (36.4°C); P = 72 beats/min; R = 22 breaths/min; and BP = 132/76 mm Hg. Twenty minutes after the transfusion started, the patient began complaining of feeling "itchy and hot" and a new rash is present on the patient's trunk. Vital signs are now T = 100.8°F (38.2°C); P = 82 beats/min; R = 24 breaths/min; BP = 146/88 mm Hg. Based on these findings, what is the priority intervention? 1) Administer an antihistamine (anti-allergenic) medication. 2) Flush the blood tubing with D5W immediately. 3) Prepare for emergency resuscitation. 4) Stop the blood transfusion immediately.

4 1 Diphenhydramine (an antihistamine) may be ordered once the physician has been notified of the patient's condition. 2 A new IV line of normal saline should be hung. 3 There is no information indicating that the patient is in danger of cardiovascular collapse or requires resuscitation. 4 The nurse should suspect a transfusion reaction. When a transfusion reaction is suspected, the infusion should be stopped immediately. The blood bag and tubing must be sent to the laboratory for analysis

While a patient is receiving hygiene care, the chest tube becomes disconnected from the water-seal chest drainage system (CDU). Which action should the nurse take immediately? 1) Clamp the chest tube close to the insertion site. 2) Set up a new drainage system and connect it to the chest tube. 3) Have the patient take and hold a deep breath while the nurse reconnects the tube to the CDU. 4) Place the disconnected end nearest the patient into a bottle of sterile water.

4 1 Do not clamp the chest tube because this can rapidly lead to a tension pneumothorax. 2 A new drainage system should be set up to decrease the risk of infection, but the immediate action is to place the disconnected end into a bottle of sterile water. 3 The new drainage system needs to be in place before asking the patient to take and hold a deep breath. 4 Recollapse of the lung can occur because of loss of negative pressure within the system. This is commonly caused by air leaks, disconnections, or cracks in the bottles or chambers. If any of these occur, the nurse should immediately place the disconnected end nearest the patient into a bottle of sterile water or saline to a depth of 2 cm to serve as an emergency water seal until a new system can be connected.

Which is the most appropriate goal for a patient with the nursing diagnosis of Deficient Fluid Volume? 1) Electrolyte balance restored, as evidenced by improved levels of alertness and cognitive orientation 2) Electrolyte balance restored, as evidenced by sodium returning to normal range 3) Patient demonstrates effective coughing and deep-breathing techniques. 4) Maintains fluid balance, as evidenced by moist mucous membranes and urinating every 4 hours

4 1 Electrolyte imbalance does not necessarily occur with Deficient Fluid Volume; if electrolyte imbalance were present, the nursing diagnosis would be different. 2 Electrolyte imbalance does not necessarily occur with Deficient Fluid Volume; if electrolyte imbalance were present, the nursing diagnosis would be different. 3 There is no evidence that this patient has a respiratory problem, so coughing and deep breathing are irrelevant. 4 Moist mucous membranes and urinating every 4 hours would demonstrate restoration of fluid balance

Chloride, bicarbonate, phosphate, and sulfate are examples of what type of charged particles and why? 1) Cations, because they carry a positive charge 2) Cations, because they carry a negative charge 3) Anions, because they carry a positive charge 4) Anions, because they carry a negative charge

4 1 Electrolytes that carry a positive charge are called cations. Cations include sodium, potassium, calcium, and magnesium. 2 Anions are electrolytes that carry a negative charge. 3 Electrolytes that carry a positive charge are called cations. 4 Anions are electrolytes that carry a negative charge; they include chloride, bicarbonate, phosphate, and sulfate

A 6-year-old child becomes upset after learning that a blood sample is needed. The child's mother scolds the child and tells him to "act your age." How should the nurse proceed? 1) Request that the mother leave the room immediately. 2) Request the help of a coworker to hold the child down. 3) Inform the child that "this won't hurt a bit." 4) Calmly approach the child and tell him what is going to happen.

4 1 Explain to the child's mother that the boy's behavior is normal. 2 The nurse may need the help of a coworker, but she should first try a calm approach. 3 Informing the child that the blood draw will not hurt is wrong and will make him distrustful of future interventions. 4 Having blood drawn may be uncomfortable and frightening for a 6-year-old child. A calm approach can alleviate some of the fear.

A client has lower extremity edema. Which intervention should the nurse implement to help with this client's venous return? 1) Increase the amount of daily exercise. 2) Wear well-fitting shoes with dry socks. 3) Adhere to prescribed medication regimen. 4) Elevate the legs above the level of the heart.

4 1 Increasing daily exercises improves arterial circulation. 2 Wearing well-fitting shoes and dry socks supports arterial circulation. 3 Adhering to prescribed medication regimen supports arterial circulation. 4 Elevating the patient's legs above the level of the heart allows gravity to promote venous return from the feet and legs.

The nurse notes that a patient's intravenous catheter site is painful, edematous, red, and warm to the touch. There is a palpable cord along the vein and the fluid infusion is sluggish. What should the nurse suspect is occurring with this patient? 1) Infiltration 2) Extravasation 3) Hematoma 4) Phlebitis

4 1 Infiltration presents as erythema, pain, and swelling. However, there is no palpable cord with inflammation. 2 Extravasation is infiltration of a vesicant substance into the tissues. Differentiating symptoms include blanching and coolness of the surrounding skin; the formation of blisters and subsequent tissue sloughing and necrosis are later signs. 3 A hematoma is a localized mass of blood outside the blood vessel. This is generally seen when a vein is nicked during an unsuccessful insertion of an IV line or when an IV line is discontinued without pressure applied over the site. 4 Phlebitis is an inflammation of the vein. It may be caused by the infusion of solutions that are irritating to the vein. Patients receiving IV solutions with potassium chloride are at a higher risk for phlebitis, as it is irritating to the vein. The symptom of a palpable cord along the vein distinguishes this as phlebitis.

A patient with underlying cardiac disease and a draining wound requires careful monitoring of fluid balance. Which method for evaluating wound drainage would be most appropriate for assessing fluid loss? 1) Draw a circle around the area of drainage on a dressing. 2) Classify drainage as less or more than the previous drainage. 3) Weigh the patient at the same time each day on the same scale. 4) Weigh dressings before they are applied and after they are removed.

4 1 Marking a circle around the wound is useful for determining the extent of drainage seeping out of a wound, but it does not provide information about how much fluid is draining. 2 Classifying the drainage as more or less is subjective and does not accurately measure the client's fluid loss. 3 Weighing the patient daily would evaluate overall fluid balance but is not sensitive to fluid loss through the wound. 4 By weighing the dressing before it is applied and after it is removed, the nurse can accurately determine the amount of drainage.

The nurse administers intravenous morphine sulfate to a patient for pain control. For which adverse effect should the nurse monitor this patient? 1) Decreased heart rate 2) Muscle weakness 3) Decreased urine output 4) Respiratory depression

4 1 Opioids do not have a major effect on heart rate. 2 Opioids do not impact muscle function. 3 Opioids do not influence urine output. 4 Opioids are potent respiratory depressants. Patients receiving opioids should be monitored for decreased rate and depth of respirations.

Which describes the passive process by which molecules of a solute move through a cell membrane from an area of higher concentration to an area of lower concentration? 1) Osmosis 2) Filtration 3) Hydrostatic pressure 4) Diffusion

4 1 Osmosis is the movement of water across a membrane from an area of a less concentrated solution to an area of more concentrated solution. 2 Filtration is the movement of water and smaller particles from an area of high pressure to one of low pressure. 3 Hydrostatic pressure is the force created by fluid within a closed system. 4 Diffusion is a passive process by which molecules move from an area of higher concentration to an area of lower concentration.

The nurse learns that a patient's wound will be sutured closed after additional healing occurs. Which type of healing is occurring with this wound? 1) Primary intention 2) Regenerative healing 3) Secondary intention 4) Tertiary intention

4 1 Primary healing occurs when a wound involves minimal or no tissue loss and has edges that are well approximated. 2 Regenerative healing takes place when a wound affects only the epidermis and dermis. No scar forms and the new (regenerated) epithelial and dermal cells form new skin that cannot be distinguished from the intact skin. Partial-thickness wounds heal by regeneration. 3 Initially the wound is allowed to heal by secondary intention, and when there is no evidence of edema, infection, or foreign matter, granulating tissue is brought together and the wound edges are sutured closed. 4 Tertiary intention is a technique used when a wound is clean contaminated or "dirty" (potentially infected)

A client's pulse oximetry reading is 90%. What action should the nurse take first? 1) Raise the head of the bed. 2) Prepare to administer oxygen. 3) Notify the healthcare provider. 4) Move the sensor to another area.

4 1 Raising the head of the bed is appropriate if the client is short of breath. There is no evidence that this client is experiencing dyspnea. 2 The client might need oxygen; however, more assessment is required. 3 The healthcare provider does not need to be notified yet. 4 The nurse should relocate the sensor to another body area. The hands may be cold or the elbow might be bent. The earlobe should be attempted before implementing any other actions.

A client developed a stage 4 pressure injury to the sacrum 6 weeks ago, and now the ulcer appears to be a shallow crater involving only partial skin loss. How should the nurse now classify this pressure injury? 1) Stage 1 pressure injury, healing 2) Stage 2 pressure injury, healing 3) Stage 3 pressure injury, healing 4) Stage 4 pressure injury, healing

4 1 Reverse staging is not done because as the ulcer heals with granulation tissue and becomes shallower, the lost muscle, subcutaneous fat, and dermis are not replaced. Pressure ulcers maintain their original staging classification throughout the healing process but are accompanied by the modifier healing. 2 Reverse staging is not done because as the ulcer heals with granulation tissue and becomes shallower, the lost muscle, subcutaneous fat, and dermis are not replaced. Pressure ulcers maintain their original staging classification throughout the healing process but are accompanied by the modifier healing. 3 Reverse staging is not done because as the ulcer heals with granulation tissue and becomes shallower, the lost muscle, subcutaneous fat, and dermis are not replaced. Pressure ulcers maintain their original staging classification throughout the healing process but are accompanied by the modifier healing. 4 Reverse staging is not done because as the ulcer heals with granulation tissue and becomes shallower, the lost muscle, subcutaneous fat, and dermis are not replaced. Pressure ulcers maintain their original staging classification throughout the healing process but are accompanied by the modifier healing.

When performing a central venous catheter dressing change, which step is correct? 1) Wear sterile gloves while removing and discarding the soiled dressing. 2) Apply pressure on the catheter-hub junction when removing the soiled dressing. 3) Place a sterile transparent dressing over the site and the catheter-hub junction. 4) Have the patient wear a mask or turn his head away from the site.

4 1 Sterile gloves should be worn when placing the new sterile dressing; however, procedure gloves are used to remove the soiled dressing. 2 The nurse should stabilize the catheter while removing the soiled dressing but not apply pressure to the catheter-hub junction. 3 The transparent dressing should cover the hub of the catheter, but not the catheter-hub junction; this makes it too difficult to remove without disturbing the integrity of the IV line or the site. 4 Aseptic technique should be used with approaching the insertion site. Therefore, both nurse and patient should wear a mask. If the patient cannot wear a mask, have him turn his head away from the insertion site during the procedure.

A patient in respiratory distress has the following arterial blood gases (ABGs) results: pH = 7.30 PCO2 = 40 HCO3 = 19 mEq/L PO2 = 80 After treatment, repeat ABGs results are pH = 7.38 PCO2 = 32 HCO3 = 19 mEq/L. What should the nurse conclude from this information? 1) Respiratory acidosis; the treatment plan is ineffective. 2) Metabolic alkalosis; the treatment plan is effective. 3) Partial compensation; the treatment plan is ineffective. 4) Full compensation; the treatment plan is effective.

4 1 The ABG is now complete compensation metabolic acidosis. 2 The ABG is now complete compensation metabolic acidosis. 3 Partial compensation would be indicated by changes in the PCO2, but the pH would still be outside the normal range. 4 Full compensation has occurred, as the PCO2 has returned the pH to the normal range. This change indicates that the treatment plan is effective.

Which is the principal system for regulation of fluid and electrolyte balance? 1) Cardiac 2) Vascular 3) Pulmonary 4) Renal

4 1 The heart is involved in fluid balance but not in electrolyte balance. 2 The vascular system is involved in fluid balance but not in electrolyte balance. 3 The pulmonary system plays a major role in regulation of acid-base balance. 4 A balance of fluid and electrolytes is essential to maintain homeostasis. Excesses or deficits can lead to severe disorders. The kidneys are the principal regulator of fluid and electrolyte balance and are the primary source of fluid output. Specific hormones (e.g., ADH, aldosterone) cause the kidneys to regulate the body's fluid and electrolyte balance.

A client asks why a pulse oximeter reading is being done when experiencing a problem with the heart. What should the nurse respond to this client? 1) "You might not have a heart problem." 2) "Heart problems are caused by breathing problems" 3) "I'm just checking it as a routine part of vital signs." 4) "The heart and lungs work together to provide oxygenation."

4 1 The nurse has no way of knowing whether the client does not have a heart problem. 2 Not all heart problems are caused by breathing problems. 3 Pulse oximetry might be a routine part of vital signs but this is not the reason why the nurse is completing this measurement. 4 The heart and lungs work together to provide oxygenation; a problem in one creates a problem in the other.

The nurse gathers the following data: BP = 150/94 mm Hg; neck veins distended; P = 104 beats/min; pulse bounding; respiratory rate = 20 breaths/min; T = 37°C (98.6°F). What disorder should the nurse suspect? 1) Hypovolemia 2) Hypercalcemia 3) Hyperkalemia 4) Hypervolemia

4 1 The patient's vital signs do not support low blood volume. 2 The patient's vital signs do not support an elevated calcium level. 3 The patient's vital signs do not support an elevated potassium level. 4 Hypervolemia results from retention of sodium and water. Blood pressure rises, the pulse is bounding, and neck veins become distended due to increased intravascular volume.

A patient has multiple open wounds that require treatment. What should the nurse do when performing dressing changes? 1) Remove all of the soiled dressings before beginning wound treatment. 2) Cleanse wounds from most contaminated to least contaminated. 3) Treat wounds on the patient's side first, then the front and back of the patient. 4) Irrigate wounds from least contaminated to most contaminated

4 1 To avoid the possibility of cross-contamination, the wound with the least amount of contamination should be treated first, progressing to the wound with the most contamination. 2 To avoid the possibility of cross-contamination, the wound with the least amount of contamination should be treated first, progressing to the wound with the most contamination. 3 To avoid the possibility of cross-contamination, the wound with the least amount of contamination should be treated first, progressing to the wound with the most contamination. 4 To avoid the possibility of cross-contamination, the wound with the least amount of contamination should be treated first, progressing to the wound with the most contamination

Which of the following interventions would help to prevent or relieve persistent nausea? 1) Assess for signs of dehydration. 2) Provide dietary supplements. 3) Have the patient sit in an upright position for 30 minutes after eating. 4) Immediately remove any food that the patient cannot eat

4 Dehydration can occur as a result of continued nausea and vomiting, so the nurse should assess for it. However, this intervention does not prevent nausea. Dietary supplements might help to prevent malnutrition. However, they do not prevent nausea; in fact, they often cause nausea. Having the patient sit upright helps to prevent respiratory aspiration should the patient vomit; it does not prevent or relieve nausea. Odors (even pleasant ones) and even the sight of food can cause nausea, so any uneaten food should be removed immediately from the room.

The nurse is assessing a patient who underwent bowel resection 2 days ago. As she auscultates the patient's abdomen, she notes low-pitched, infrequent bowel sounds. How should she document this finding? 1) Hyperactive bowel sounds 2) Abdominal bruit sounds 3) Normal bowel sounds 4) Hypoactive bowel sounds

4 Hypoactive bowel sounds are low-pitched, infrequent, and quiet. An abdominal bruit is a hollow, blowing sound found over an artery, such as the iliac artery. Normal bowel sounds are high-pitched with approximately 5 to 35 gurgles occurring every minute. Hyperactive bowel sounds are very high pitched and more frequent than normal bowel sounds.

A patient has anemia. An appropriate goal for that the patient would be for him to increase his intake of which nutrient? 1) Calcium 2) Magnesium 3) Potassium 4) Iron

4 Iron deficiency causes anemia; therefore, the nurse should encourage the patient with anemia to increase his intake of iron. Increasing calcium intake helps prevent osteoporosis. Magnesium supplementation may decrease the risk of hypertension and coronary artery disease in women. Potassium is essential for muscle contraction, acidbase balance, and blood pressure control.

After instructing a mother about nutrition for a preschool-age child, which statement by the mother would indicate correct understanding of the topic? 1) I usually use dessert only as a reward for eating other foods. 2) I will hide vegetables in casseroles and stews to get my child to eat them. 3) I do not give my child snacks; they simply spoil his appetite for meals. 4) I know that lifelong food habits are developed during this stage of life

4 Lifelong food habits are developed during the preschool stage of life. Therefore, the mother should widen the variety of foods she introduces to her child. Desserts should not be used as rewards for eating other foods. This practice can shape an attitude about food that can lead to eating disorders later in life. Preschool-age children often refuse combined foods such as casseroles and stews. Because they are active, preschoolers require nutritious between-meal snacks.

Which collaborative interventions will help prevent paralytic ileus in a patient who underwent right hemicolectomy for colon cancer? 1) Administering morphine 4 mg intravenously every 2 hours for pain 2) Administering IV fluids at 125 ml/hr 3) Inserting an indwelling urinary catheter to monitor I and O 4) Keeping the patient NPO until bowel sounds return

4 Patients who require bowel surgery typically remain NPO until peristalsis returns, helping to prevent paralytic ileus, a complication that can occur after the bowel is surgically manipulated. Administering morphine promotes comfort but may increase the risk of ileus. Administering IV fluids prevents dehydration, but does not directly prevent ileus. Inserting an indwelling urinary catheter prevents urine retention and facilitates monitoring postoperative urine output.

The nurse has taught a patient about the primary risk factors for irritable bowel syndrome. Which behavior by the patient would be evidence of learning? The patient: 1) Reduces her intake of gluten-containing products. 2) Does not consume foods that contain lactose. 3) Consumes only 4 cups of caffeinated coffee per day. 4) Takes measures to reduce her stress level.

4 Stress is a primary factor in the development of irritable bowel syndrome. Other risk factors include caffeine consumption and lactose intolerance; however, they are not primary risk factors. Celiac disease is associated with gluten intake.

Which organ relies almost exclusively on glucose for energy? 1) Liver 2) Heart 3) Pancreas 4) Brain

4 The brain relies almost exclusively on glucose for energy. The heart, pancreas, and liver do no

A patient complains that she passes urine whenever she sneezes or coughs. How should the nurse document this complaint in the patients healthcare record? 1) Transient incontinence 2) Overflow incontinence 3) Urge incontinence 4) Stress incontinence

4 Stress incontinence is an involuntary loss of urine that occurs with increased intra-abdominal pressure. Activities that typically produce the symptom include sneezing, coughing, laughing, lifting, and exercise. Transient incontinence is a short-term incontinence that is expected to resolve spontaneously. It is typically caused by urinary tract infection or medications, such as diuretics. Overflow incontinence is the loss of urine when the bladder becomes distended; it is commonly associated with fecal impaction, enlarged prostate, and neurological conditions. Urge incontinence is the involuntary loss of urine associated with a strong urge to void.

A patient is admitted with high BUN and creatinine levels, low blood pH, and elevated serum potassium level. Based on these laboratory findings the nurse suspects which diagnosis? 1) Cystitis 2) Renal calculi 3) Enuresis 4) Renal failure

4 Elevated BUN, creatinine, and serum potassium levels and low blood pH are signs of renal failure. Cystitis is an infection of the bladder and would not result in abnormal renal function. Renal calculi typically produce blood in the urine but do not lead to marked renal dysfunction and failure. Enuresis is involuntary urination, particularly common in children, and does not produce renal dysfunction. The cause of enuresis is often emotional, developmental, or trauma related.

Which intervention should the nurse take first to promote micturition in a patient who is having difficulty voiding? 1) Insert an indwelling urinary catheter. 2) Notify the provider immediately. 3) Insert a straight catheter. 4) Pour warm water over the patients perineum.

4 The nurse should perform independent nursing measures, such as pouring warm water over the patients perineum before notifying the provider. If nursing measures fail, the nurse should notify the provider. The provider may order an indwelling urinary catheter or a straight catheter to relieve the patients urinary retention.

A nurse is administering an enema medicated with sodium polystyrene sulfonate (Kayexalate) to an older adult patient who has hyperkalemia. The nurse should insert the tip of the rectal tube.... ANSWER IS A MEASUREMENT

7.5 cm to 10 cm (3 to 4 in) in

A nurse admits a 78 y/o pt who has COPD and is coughing thick green sputum for 4 days. Oxygen saturation is 88% on room air. What type of oxygen equipment should the nurse anticipate the provider will order? A. Venturi mask B. Non rebreather mask C. Face tent D. Face mask

A

A nurse assesses a stage IV pressure injury and notes a large amount of exudate soaking the dressing and the periwound. What type of dressing should the nurse ask the provider to order? A. Alginate B. Hydrocolloid C. Wet to damp D. Dry gauze

A

A nurse assesses an adult pt whose BMI is 17 and reports recent weight loss over the passed couple of months. What physical assessment findings should the nurse expect? A. Thin extremities and excess skin folds B. Kyphosis C. Heart murmur D. Elevated temperature

A

A nurse examines a patient and notes several ecchymotic areas with intact skin on the patient's arms and legs. How should the nurse document this finding? A. Contusion B. Tunnel C. Abrasion D. Laceration

A

A nurse is caring for a patient who is post operative knee replacement. How should the nurse expect the wound to heal? A. Primary intention B. Tertiary intention C. Regenerative healing D. Secondary intention

A

A nurse is planning care for a pt who has community acquired pneumonia. What intervention should the nurse include in the plan? A. Increase fluid intake to thin secretions B. Limit exercise to 30 minutes walking a day C. Wear a mask when going shopping D. Check temp q2h

A

A nurse is planning care for a pt whose gag reflex is diminished and has difficulty swallowing after a stroke. What intervention should the nurse include in the plan? A. Check the inside of the pt's mouth for pocketing of food B. Encourage sips of fluid between each meals C. Offer a variety of cool or cold foods D. Provide finger foods the pt can eat independently

A

A nurse is preparing to administer oxygen therapy to several pts. Which pt will require humidified oxygen? A. Oxygen at 4L/min NC B. Oxygen via venturi mask at 24% fio2 C. Oxygen at 2L/min NC D. Oxygen via venturi mask at 50% fio2

A

A nurse is working in the ER when a pt is brought in by friends after reporting sudden right sided chest pain and SOB. What type of assessment should the nurse perform? A. Focused B. Comprehensive C. Functional D. Ongoing

A

A nurse takes a health history from a patient who reports a 2 week history of feelings full after eating about half of each meal. What further assessment should the nurse do? A. Check the serum albumin B. Measure the blood pressure and pulse lying and standing C. Inspect the oral cavity D. Calculate the BMI

A

The RN asks the nursing student working them to discuss the administration of oxygen. Which of the following statements indicates a need for further education? A. "The administration of oxygen is considered a nursing intervention and does not need a prescription" B. "When placing a nasal cannula, the tubing should wrap behind each ear and secure under the chin" C. "When placing a face mask, it should be cover from the bridge of the nose to under the chin" D. "It is important to monitor for skin breakdown behind the ears, under the chin and on the nose"

A

The nurse assesses respiration, heart rate, pulse oximetry and notes the following: RR 34, HR 120, and pulse ox 89%. How should the nurse document this? A. Tachypnea B. Bradycardia C. Kussmauls respirations D. Cheyne stokes respirations

A

A nurse is preparing to insert an indwelling urinary catheter for a female patientt. When beginning the insertion procedure, the nurse should instruct the patient to: A) bear down. B) take deep breaths. C) sip water. D) hold her breath.

A A - Bearing down as if to void relaxes the external sphincter and aids in the insertion procedure. This is the appropriate instruction for the patient. B - It is not necessary for the patient to take deep breaths during a catheter insertion. C- It is not necessary for the patient to sip water during a catheter insertion. D - It is not necessary for the patient to hold her breath during a catheter insertion

A nurse assesses a pt who has suddenly become restless. The nurse notes nasal laying and use of accessory muscles to breathe. What other findings should the nurse expect? SATA A. Tachypnea B. Decreased blood pressure C. Tachycardia D. Moist mucous membranes E. Clear lung sounds

A C

A nurse admits a 90 y o woman from a long term care facility after a fall during which the pt sustained a fractured femur. The pt is prescribed bed rest, a mild opioid q4h and a regular diet. What factors should the nurse identify will affect the pt's bowel function? SATA A. Age B. Diet order C. Gender D. Bed rest E. Opioid medication

A D E

A nurse is preparing to administer a cleansing enema to a patient who is prone to fecal incontinence due to poor sphincter control and is unlikely to retain the enema solution. Which of the following interventions is appropriate for this patient? A) Place the patient in the dorsal recumbent position on a bedpan. B) Administer the enema while the patient sits on the toilet. C) Administer an antidiarrheal med 3 hr prior to the enema. D) Instil 200 mL of fluid at 15-min intervals times four.

A) Place the patient in the dorsal recumbent position on a bedpan.

A nurse is preparing to administer the first of two large volume, cleansing enemas prescribed for a patient in preparation for a diagnostic procedure. Which of the following is an appropriate step in the procedure? A) Warm the enema solution prior to instillation B) Prepare 1,500mL of enema fluid C) Use tap water as the enema fluid D) Hang the enema container 24 inchs above the anus

A) Warm the enema solution prior to instillation

A nurse is caring for a patient who is 8 hours post operative abdominal surgery and their diet order has been changed from NPO to clear liquid. What care can the nurse delegate to the nursing assistive personnel ? SATA A. monitoring the intravenous fluid B. Teaching the patient how to deep breathe and cough C. Assisting the patient into a semi fowler position to drink apple juice D. Documenting the amount of oral intake on the fluid balance sheet E. Changing the dressing

A, C, D

A clinical instructor observes a nursing student administering a cleaning enema to a patient. What action if done by the student should the instructor identify as incorrect ? SATA A. The patient is in the semi-fowlers position B. The patient is draped with a bath blanket so that only buttocks and rectum are exposed C. The container is hanging 2 feet about the level of the patients hips D. The student is wearing clean gloves E. A waterproof pad is under the patients buttocks

A,C

A patient who has bladder cancer tells the nurse that, of the various urinary diversion options the surgeon presented, she prefers one that will allow her to have some control over urinary elimination. The nurse should explain the option that will allow that is A. A Kock's pouch. B. An ileal conduit. C. Acutaneous ureterostomy. D. A nephrostomy.

A. A Kock's pouch This is a continent ileal bladder conduit that does not require an external drainage collection device because the patient self-catheterizes every 2-4 hours to remove urine. This device will provide the control the patient desires.

A nurse is obtaining health history from a young adult patient who has a colostomy. The patient reports frequent episodes of loose stools over the last month, but has no signs of infection or bowel obstruction. He reports that his concerns about leakage have limited his social activities. Which of the following should the nurse recommend? A. Consume foods that are low in fiber content. B. Take an ounce of mineral oil twice a day. C. Add buttermilk and cranberry juice to the diet. D. Increase water intake to 3 to 3.5 L per day.

A. Consume foods that are low in fiber content. Foods low in fiber help thicken the stool; examples include rice, noodles, white bread, cream cheese, lean meats, fish, and poultry.

A nurse is planning care for a patient who has a stress incontinence. What intervention should the nurse include in the plan ? A. Instruct in pelvic floor muscle exercises B. Teach intermittent straight catheterization C. Provide a bedside commode D. Establish a scheduled voiding routine

A. Do kegel exercises to strengthen the pelvic muscle

The RN is reviewing a plan of care for a 7 year old with nocturnal enuresis. Which of the following statements by the parents indicates understanding: A. "This is caused by a deficiency in ADH and my child will grow out of it" B. "My child should see a behavior health specialist as this is an indication of a major cognitive delay" C. "My child should be given medication daily to prevent this from continuing" D. "My child is just being defiant and punishing or shaming them will make it stop"

A. Nocturnal enuresis is bed wetting 10% of children from 5-10 wet the bed and mostly common with boys.

A home care nurse makes a visit to a house where 2 families live. 1 child has a temp of 102F/39.1C and is coughing. What action should the nurse take? SATA A. Give the child additional fluids B. Allow the child to rest C. Limit time spent with other children for play to 3 hours a day D. Teach the mother how to perform chest percussion E. Ask the provider to order oxygen therapy for the child.

AB

The RN asks the nursing student working with them to describe the process of monitoring oxygen saturation. Which of the following responses indicates an understanding of this process? SELECT ALL THAT APPLY A. "Pulse oximeter measurement can be performed by the NAP, LPN or RN" B. "Before placing the probe, you should check that there is adequate capillary refill to the site" C. "There is no need to remove nail polish from fingernails, it does not affect accuracy of the reading" D. "I will record the date and time of each reading obtained on the flow sheet" E. "Once on oxygen, I no longer need to assess the client's breath sounds"

ABD, maybe C

Which urine specific gravity would be expected in a patient admitted with dehydration? 1) 1.002 2) 1.010 3) 1.025 4) 1.030

ANS: 4 Normal urine specific gravity ranges from 1.010 to 1.025. Specific gravity less than 1.010 indicates fluid volume excess, such as when the patient has fluid overload (too much IV fluid) or when the kidneys fail to concentrate urine. Specific gravity greater than 1.025 is a sign of deficient fluid volume that occurs, for example, as a result of blood loss or dehydration.

A nurse should recognize which of the following is correct regarding albumin level as a diagnostic marker for nutritional status?

Albumin level is a poor short-term indicator of protein status. (Albumin is not sensitive to acute changes in nutritional status. Its long half-life (21 days) makes it a better indicator of chronic illness states than of current protein status at a given point in time

An RN is observing an LPN insert an indwelling urinary catheter into a female client, which of the following activities would the RN need to intervene on? A. Places the client in the left lateral lying position B. Wipes from clitoris to anus cleansing the urinary meatus last C. Avoids moving across the sterile field with used materials D. Inserts the urinary catheter a total of 3-5 inches (7.5-12.5cm)

Apple

The RN is speaking with the mother of a two year old at a well child visit. The mother expresses concern that the child is not yet toilet trained. Which of the following responses by the RN is correct? A. "This is normal, most children toilet train between 18-36 months" B. "This is abnormal and you should consider evaluation for developmental delay" C. "Your child should be bowel trained by 24 months and bladder trained by 36 months" D. "Your child may be experiencing an imbalance in ADH and should be evaluated by endocrinology"

Apple

The RN is formulating a care plan for a 35 year old male with a BMI of 38 and documented lifestyle including no formal exercise, enjoys video games and eats almost exclusively takeout. With this information only what would be appropriate nursing goals/outcomes for this client? SELECT ALL THAT APPLY A. Progressively loses weight toward desired goal over 6 months B. Incorporates physical activities into daily life C. Stops playing video games D. Increases self-esteem over three months E. States factors contributing to weight in daily life

Apple Boy Elephant

A nurse is preparing to administer an oil-retention enema to a patient who has constipation. The nurse explains that the patient should try to retain the instilled oil for? ANSWER IS A TIME RANGE :)

At least 30 min, but preferably as long as he can

A home care nurse is providing wound care for a patient who experienced wound dehiscence in the post operative period and now has a full thickness wound. What finding would indicate a complication of wound healing? A. Beefy red tissue, which is easily damaged. B. Increase in wound depth and purulent drainage. C. Epithelial tissue at wound edges. D. Moderate amount of serosanguinous exudate.

B

A nurse admits a frail 85 y o woman who is incontinent of urine and bed bound from a long term care facility. The pt has a UTI and only oriented to person. When planning care, what intervention should the nurse include in the plan? A. Teach the pt to deep breathe and cough B. Use absorbent products to wick moisture from the skin C. Provide small frequent meals D. Wash perineal area with hot water and soap after each episode of incontinence

B

A nurse admits a frail older patient from a long term care facility after a fall during which the patient sustained a fractured femur. The patient is bed bound after surgical repair. What additional assessment should the nurse perform? A. Morse B. Braden C. Katz D. Lawton

B

A nurse assesses a patient's sacrum and notes an 3 x 4 cm nonblanchable reddened area. When planning care what intervention should the nurse include in the plan? A. Place a donut-type device under the patient's sacrum B. Turn and reposition patient every two hours C. Inspect sacral area every three days D. Massage sacral area twice daily with lotion

B

A nurse is planning care for a pt who has acute diarrhea after eating contaminated food. What intervention should the nurse include in the plan? A. Check capillary refill B. Monitor stool frequency and volume C. Increase intake of high fiber food D. Monitor for development of hemorrhoids

B

A nurse is preparing to collect a wound culture using the Levine technique. How should the nurse proceed? A. Remove the dressing and swab the edge of the wound B. Press the swab against the granulating tissue and rotate C. Roll the swab across the exudate in the wound bed D. Swab several different areas of the wound bed

B

A nurse is teaching an older adult who has osteoporosis about food choices to increase calcium intake. What suggestions should the RN make? A. Exchange butter for olive oil when cooking B. Add broccoli to salad each day C. Switch from meat and potatoes to rice and beans D. Include 1 serving of fruit with each meal

B

A nurse takes a health history from a pt who reports a 2 week hx of feeling full after eating less than 1/2 their meals. What further assessment should the nurse do? A. Measure the abdominal circumference B. A three day food record C. Auscultate bowel sounds D. Calculate the waist to hip ratio

B

The RN is giving a report on a client over the phone to the Nurse Practitioner on call using SBAR. Which of the following statements should be last? A. "Mr. Garcia's wound is erythematous with new purulent discharge." B. "I think we need to do a culture and consider an anti-infective treatment" C. "Mr. Garcia is a 56 year old male with uncontrolled diabetes with a laceration healing by secondary intention" D. "His wound continues to not improve and appears it may now be infected"

B

Which of the following actions should the nurse take when removing a patient's indwelling urinary catheter? A) Pull the catheter out as quickly as possible. B) Deflate the balloon completely before removal. C) Make sure the patient has voided within 12 hr post removal. D) Tell the patient to expect to feel a tugging sensation on removal.

B A - A slow, steady motion is recommended for urinary catheter removal. B - If any inflation solution remains in the balloon, trauma to the urethral canal is likely with removal of the catheter. C - If the patient does not void within 6 to 8 hr of removal, it is often necessary to recatheterize the bladder. It is inappropriate to wait up to 12 hr for the patient to void. D- It is common for patients to feel a burning sensation as a urinary catheter is removed, but there should not be any tugging involved.)

A nurse who is preparing to insert a straight urinary catheter for a male patient should A) grasp the penis at its base. B) apply light traction to the penis. C) hold the penis parallel to the patient's body. D) lift the penis to a 45° angle to the patient's body.

B A - Grasping the penis at its base would not effectively straighten the urethral canal to ease catheter insertion. B - Lifting the penis to a position perpendicular to the body while applying light traction straightens the urethral canal to facilitate catheter insertion. C - Holding the penis parallel to the patient's body would not effectively straighten the urethral canal to ease catheter insertion. D - Lifting the penis to a 45° angle to the patient's body would not effectively straighten the urethral canal to ease catheter insertion

A nurse who is administering a return-flow enema to a patient should instill 100 mL of enema fluid and then: A) instruct the patient to retain the fluid B) lower the container to allow the solution to flow back out C) help the patient to the toilet or bedside commode D) wait 5min and instill another 100mL of fluid

B) lower the container to allow the solution to flow back out


Kaugnay na mga set ng pag-aaral

Series 65 Unit 2 - Insurance Based Products

View Set

Demonstration and Verbal Instructions

View Set

American Government the Purpose of Government

View Set

Section 1-3 Square Roots Properties

View Set

Real Estate Principles - Chapter Four; Transferring Real Estate

View Set

EASA ATPL HPL Chapter 6 "Visual Systems"

View Set

Transforming and distorting objects

View Set

Chapter 1 = Chemistry: The Science of Change

View Set

Training and Development: Chapter 8

View Set

Econ 211 vocab and definition review (test 2)

View Set

UNIT 1: GET READY TO USE TECHNOLOGY IN THE CLASSROOM

View Set

Topic 10: Muscular System Gross Anatomy

View Set

Missed Questions - Health Guarantee Exam

View Set

AFI's 100 Greatest Movie Quotes of All Time

View Set